119 Fundamentals of Mathematics 1

Fundamentals of Mathematics

By:

Denny Burzynski

Wade Ellis


Fundamentals of Mathematics

By:

Denny Burzynski

Wade Ellis

Online:

< http://cnx.org/content/col10615/1.4/ >

C O N N E X I O N S

Rice University, Houston, Texas

This selection and arrangement of content as a collection is copyrighted by Denny Burzynski, Wade Ellis. It is licensed under the Creative Commons Attribution 2.0 license (http://creativecommons.org/licenses/by/2.0/).

Collection structure revised: August 18, 2010

PDF generated: July 29, 2013

For copyright and attribution information for the modules contained in this collection, see p. 699.

Table of Contents

Preface . . . . . . . . . . . . . . . . . . . . . . . . . . . . . . . . . . . . . . . . . . . . . . . . . . . . . . . . . . . . . . . . . . . . . . . . . . . . . . . . . . . . . . . . . . . . . . . 1

Acknowledgements . . . . . . . . . . . . . . . . . . . . . . . . . . . . . . . . . . . . . . . . . . . . . . . . . . . . . . . . . . . . . . . . . . . . . . . . . . . . . . . . . . 7

1 Addition and Subtraction of Whole Numbers

1.1 Objectives . . . . . . . . . . . . . . . . . . . . . . . . . . . . . . . . . . . . . . . . . . . . . . . . . . . . . . . . . . . . . . . . . . . . . . . . . . . . . . . . . . . 9

1.2 Whole Numbers . . . . . . . . . . . . . . . . . . . . . . . . . . . . . . . . . . . . . . . . . . . . . . . . . . . . . . . . . . . . . . . . . . . . . . . . . . . . 10

1.3 Reading and Writing Whole Numbers . . . . . . . . . . . . . . . . . . . . . . . . . . . . . . . . . . . . . . . . . . . . . . . . . . . . . . 15

1.4 Rounding Whole Numbers . . . . . . . . . . . . . . . . . . . . . . . . . . . . . . . . . . . . . . . . . . . . . . . . . . . . .. . . . . . . . . . . . . 21

1.5 Addition of Whole Numbers . . . . . . . . . . . . . . . . . . . . . . . . . . . . . . . . . . . . . . . . . . . . . . . . . . . . . . . . . . . . . . . . 29

1.6 Subtraction of Whole Numbers . . . . . . . . . . . . . . . . . . . . . . . . . . . . . . . . . . . . . . . . . . . . . . . . . . . . . . . . . . . . . 44

1.7 Properties of Addition . . . . . . . . . . . . . . . . . . . . . . . . . . . . . . . . . . . . . . . . . . . . . . . . . . . . . . . . . . . . . . . . . . . . . . 62

1.8 Summary of Key Concepts . . . . . . . . . . . . . . . . . . . . . . . . . . . . . . . . . . . . . . . . . . . . . . . . . . . . . . . . . . . . . . . . . 68

1.9 Exercise Supplement . . . . . . . . . . . . . . . . . . . . . . . . . . . . . . . . . . . . . . . . . . . . . . . . . . . . . . . . . . . . . . . . . . . . . . . 69

1.10 Prociency Exam . . . . . . . . . . . . . . . . . . . . . . . . . . . . . . . . . . . . . . . . . . . . . . . . . . . . . . . . . . . . . . . . . . . . . . . . . 74

Solutions . . . . . . . . . . . . . . . . . . . . . . . . . . . . . . . . . . . . . . . . . . . . . . . . . . . . . . . . . . . . . . . . . . . . . . . . . . . . . . . . . . . . . . . . 76

2 Multiplication and Division of Whole Numbers

2.1 Objectives . . . . . . . . . . . . . . . . . . . . . . . . . . . . . . . . . . . . . . . . . . . . . . . . . . . . . . . . . . . . . . . . . . . . .. . . . . . . . . . . . . 91

2.2 Multiplication of Whole Numbers . . . . . . . . . . . . . . . . . . . . . . . . . . . . . . . . . . . . . . . . . . . . . .. . . . . . . . . . . . . 92

2.3 Concepts of Division of Whole Numbers . . . . . . . . . . . . . . . . . . . . . . . . . . . . . . . . . . . . . . . . . . . . . . . . . . . 105

2.4 Division of Whole Numbers . . . . . . . . . . . . . . . . . . . . . . . . . . . . . . . . . . . . . . . . . . . . . . . . . . . .. . . . . . . . . . . . 113

2.5 Some Interesting Facts about Division . . . . . . . . . . . . . . . . . . . . . . . . . . . . . . . . . . . . . . . . . . . . . . . . . . . . . 126

2.6 Properties of Multiplication . . . . . . . . . . . . . . . . . . . . . . . . . . . . . . . . . . . . . . . . . . . . . . . . . . . . . . . . . . . . . . . 130

2.7 Summary of Key Concepts . . . . . . . . . . . . . . . . . . . . . . . . . . . . . . . . . . . . . . . . . . . . . . . . . . . . . . . . . . . . . . . . 136

2.8 Exercise Supplement . . . . . . . . . . . . . . . . . . . . . . . . . . . . . . . . . . . . . . . . . . . . . . . . . . . . . . . . . . . . . . . . . . . . . . 137

2.9 Prociency Exam . . . . . . . . . . . . . . . . . . . . . . . . . . . . . . . . . . . . . . . . . . . . . . . . . . . . . . . . . . . . . .. . . . . . . . . . . . 140

Solutions . . . . . . . . . . . . . . . . . . . . . . . . . . . . . . . . . . . . . . . . . . . . . . . . . . . . . . . . . . . . . . . . . . . . . . . . . . . . . . . . . . . . . . . 142

3 Exponents, Roots, and Factorization of Whole Numbers

3.1 Objectives . . . . . . . . . . . . . . . . . . . . . . . . . . . . . . . . . . . . . . . . . . . . . . . . . . . . . . . . . . . . . . . . . . . . .. . . . . . . . . . . . 153

3.2 Exponents and Roots . . . . . . . . . . . . . . . . . . . . . . . . . . . . . . . . . . . . . . . . . . . . . . . . . . . . . . . . . .. . . . . . . . . . . . 154

3.3 Grouping Symbols and the Order of Operations . . . . . . . . . . . . . . . . . . . . . . . . . . . . . . . . . . . . . . . . . . . 162

3.4 Prime Factorization of Natural Numbers . . . . . . . . . . . . . . . . . . . . . . . . . . . . . . . . . . . . . . .. . . . . . . . . . . . 172

3.5 The Greatest Common Factor . . . . . . . . . . . . . . . . . . . . . . . . . . . . . . . . . . . . . . . . . . . . . . . . . . . . . . . . . . . . . 180

3.6 The Least Common Multiple . . . . . . . . . . . . . . . . . . . . . . . . . . . . . . . . . . . . . . . . . . . . . . . . . . . . . . . . . . . . . . 184

3.7 Summary of Key Concepts . . . . . . . . . . . . . . . . . . . . . . . . . . . . . . . . . . . . . . . . . . . . . . . . . . . . . . . . . . . . . . . . 191

3.8 Exercise Supplement . . . . . . . . . . . . . . . . . . . . . . . . . . . . . . . . . . . . . . . . . . . . . . . . . . . . . . . . . . . . . . . . . . . . . . 193

3.9 Prociency Exam . . . . . . . . . . . . . . . . . . . . . . . . . . . . . . . . . . . . . . . . . . . . . . . . . . . . . . . . . . . . . .. . . . . . . . . . . . 198

Solutions . . . . . . . . . . . . . . . . . . . . . . . . . . . . . . . . . . . . . . . . . . . . . . . . . . . . . . . . . . . . . . . . . . . . . . . . . . . . . . . . . . . . . . . 200

4 Introduction to Fractions and Multiplication and Division of Fractions 4.1 Objectives . . . . . . . . . . . . . . . . . . . . . . . . . . . . . . . . . . . . . . . . . . . . . . . . . . . . . . . . . . . . . . . . . . . . .. . . . . . . . . . . . 213

4.2 Fractions of Whole Numbers . . . . . . . . . . . . . . . . . . . . . . . . . . . . . . . . . . . . . . . . . . . . . . . . . . .. . . . . . . . . . . . 214

4.3 Proper Fractions, Improper Fractions, and Mixed Numbers . . . . . . . . . . . . . . . . . . . . . . . . . . . . . . . . 223

4.4 Equivalent Fractions, Reducing Fractions to Lowest Terms, and Raising Fractions to Higher Terms . . . . . . . . . . . . . . . . . . . . . . . . . . . . . . . . . . . . . . . . . . . . . . . . . . . . . . . . . . . . . . . . . . . . . . . . . . . 233

4.5 Multiplication of Fractions . . . . . . . . . . . . . . . . . . . . . . . . . . . . . . . . . . . . . . . . . . . . . . . . . . . . . . . . . . . . . . . . 244

4.6 Division of Fractions . . . . . . . . . . . . . . . . . . . . . . . . . . . . . . . . . . . . . . . . . . . . . . . . . . . . . . . . . . . . . . . . . . . . . . 255

4.7 Applications Involving Fractions . . . . . . . . . . . . . . . . . . . . . . . . . . . . . . . . . . . . . . . . . . . . . . .. . . . . . . . . . . . 261

4.8 Summary of Key Concepts . . . . . . . . . . . . . . . . . . . . . . . . . . . . . . . . . . . . . . . . . . . . . . . . . . . . . . . . . . . . . . . . 268

4.9 Exercise Supplement . . . . . . . . . . . . . . . . . . . . . . . . . . . . . . . . . . . . . . . . . . . . . . . . . . . . . . . . . . . . . . . . . . . . . . 270

iv

4.10 Prociency Exam . . . . . . . . . . . . . . . . . . . . . . . . . . . . . . . . . . . . . . . . . . . . . . . . . . . . . . . . . . . . . . . . . . . . . . . . 276

Solutions . . . . . . . . . . . . . . . . . . . . . . . . . . . . . . . . . . . . . . . . . . . . . . . . . . . . . . . . . . . . . . . . . . . . . . . . . . . . . . . . . . . . . . . 278

5 Addition and Subtraction of Fractions, Comparing Fractions, and Complex Fractions 5.1 Objectives . . . . . . . . . . . . . . . . . . . . . . . . . . . . . . . . . . . . . . . . . . . . . . . . . . . . . . . . . . . . . . . . . . . . .. . . . . . . . . . . . 295

5.2 Addition and Subtraction of Fractions with Like Denominators . . . . . . . . . . . . . . . . . . . . . . . . . . . . 296

5.3 Addition and Subtraction of Fractions with Unlike Denominators . . . . . . . . . . . . . . . . . . . . . . . . . . 300

5.4 Addition and Subtraction of Mixed Numbers . . . . . . . . . . . . . . . . . . . . . . . . . . . . . . . . . . . . . . . . . . . . . . 306

5.5 Comparing Fractions . . . . . . . . . . . . . . . . . . . . . . . . . . . . . . . . . . . . . . . . . . . . . . . . . . . . . . . . . . . . . . . . . . . . . . 310

5.6 Complex Fractions . . . . . . . . . . . . . . . . . . . . . . . . . . . . . . . . . . . . . . . . . . . . . . . . . . . . . . . . . . . . . . . . . . . . . . . . 314

5.7 Combinations of Operations with Fractions . . . . . . . . . . . . . . . . . . . . . . . . . . . . . . . . . . . . . . . . . . . . . . . . 318

5.8 Summary of Key Concepts . . . . . . . . . . . . . . . . . . . . . . . . . . . . . . . . . . . . . . . . . . . . . . . . . . . . . . . . . . . . . . . . 323

5.9 Exercise Supplement . . . . . . . . . . . . . . . . . . . . . . . . . . . . . . . . . . . . . . . . . . . . . . . . . . . . . . . . . . . . . . . . . . . . . . 323

5.10 Prociency Exam . . . . . . . . . . . . . . . . . . . . . . . . . . . . . . . . . . . . . . . . . . . . . . . . . . . . . . . . . . . . . . . . . . . . . . . . 327

Solutions . . . . . . . . . . . . . . . . . . . . . . . . . . . . . . . . . . . . . . . . . . . . . . . . . . . . . . . . . . . . . . . . . . . . . . . . . . . . . . . . . . . . . . . 329

6 Decimals

6.1 Objectives . . . . . . . . . . . . . . . . . . . . . . . . . . . . . . . . . . . . . . . . . . . . . . . . . . . . . . . . . . . . . . . . . . . . .. . . . . . . . . . . . 339

6.2 Reading and Writing Decimals . . . . . . . . . . . . . . . . . . . . . . . . . . . . . . . . . . . . . . . . . . . . . . . . . . . . . . . . . . . . 340

6.3 Converting a Decimal to a Fraction . . . . . . . . . . . . . . . . . . . . . . . . . . . . . . . . . . . . . . . . . . . .. . . . . . . . . . . . 346

6.4 Rounding Decimals . . . . . . . . . . . . . . . . . . . . . . . . . . . . . . . . . . . . . . . . . . . . . . . . . . . . . . . . . . . .. . . . . . . . . . . . 350

6.5 Addition and Subtraction of Decimals . . . . . . . . . . . . . . . . . . . . . . . . . . . . . . . . . . . . . . . . . . . . . . . . . . . . . 354

6.6 Multiplication of Decimals . . . . . . . . . . . . . . . . . . . . . . . . . . . . . . . . . . . . . . . . . . . . . . . . . . . . .. . . . . . . . . . . . 360

6.7 Division of Decimals . . . . . . . . . . . . . . . . . . . . . . . . . . . . . . . . . . . . . . . . . . . . . . . . . . . . . . . . . . .. . . . . . . . . . . . 371

6.8 Nonterminating Divisions . . . . . . . . . . . . . . . . . . . . . . . . . . . . . . . . . . . . . . . . . . . . . . . . . . . . . .. . . . . . . . . . . . 383

6.9 Converting a Fraction to a Decimal . . . . . . . . . . . . . . . . . . . . . . . . . . . . . . . . . . . . . . . . . . . .. . . . . . . . . . . . 388

6.10 Combinations of Operations with Decimals and Fractions . . . . . . . . . . . . . . . . . . . . .. . . . . . . . . . . . 394

6.11 Summary of Key Concepts . . . . . . . . . . . . . . . . . . . . . . . . . . . . . . . . . . . . . . . . . . . . . . . . . . . . . . . . . . . . . . . 398

6.12 Exercise Supplement . . . . . . . . . . . . . . . . . . . . . . . . . . . . . . . . . . . . . . . . . . . . . . . . . . . . . . . . . . . . . . . . . . . . . 399

6.13 Prociency Exam . . . . . . . . . . . . . . . . . . . . . . . . . . . . . . . . . . . . . . . . . . . . . . . . . . . . . . . . . . . . . . . . . . . . . . . . 402

Solutions . . . . . . . . . . . . . . . . . . . . . . . . . . . . . . . . . . . . . . . . . . . . . . . . . . . . . . . . . . . . . . . . . . . . . . . . . . . . . . . . . . . . . . . 404

7 Ratios and Rates

7.1 Objectives . . . . . . . . . . . . . . . . . . . . . . . . . . . . . . . . . . . . . . . . . . . . . . . . . . . . . . . . . . . . . . . . . . . . .. . . . . . . . . . . . 419

7.2 Ratios and Rates . . . . . . . . . . . . . . . . . . . . . . . . . . . . . . . . . . . . . . . . . . . . . . . . . . . . . . . . . . . . . . . . . . . . . . . . . . 419

7.3 Proportions . . . . . . . . . . . . . . . . . . . . . . . . . . . . . . . . . . . . . . . . . . . . . . . . . . . . . . . . . . . . . . . . . . . . . . . . . . . . . . . 425

7.4 Applications of Proportions . . . . . . . . . . . . . . . . . . . . . . . . . . . . . . . . . . . . . . . . . . . . . . . . . . . .. . . . . . . . . . . . 431

7.5 Percent . . . . . . . . . . . . . . . . . . . . . . . . . . . . . . . . . . . . . . . . . . . . . . . . . . . . . . . . . . . . . . . . . . . . . . . .. . . . . . . . . . . . 437

7.6 Fractions of One Percent . . . . . . . . . . . . . . . . . . . . . . . . . . . . . . . . . . . . . . . . . . . . . . . . . . . . . . . . . . . . . . . . . . 444

7.7 Applications of Percents . . . . . . . . . . . . . . . . . . . . . . . . . . . . . . . . . . . . . . . . . . . . . . . . . . . . . . .. . . . . . . . . . . . 448

7.8 Summary of Key Concepts . . . . . . . . . . . . . . . . . . . . . . . . . . . . . . . . . . . . . . . . . . . . . . . . . . . . . . . . . . . . . . . . 459

7.9 Exercise Supplement . . . . . . . . . . . . . . . . . . . . . . . . . . . . . . . . . . . . . . . . . . . . . . . . . . . . . . . . . . . . . . . . . . . . . . 460

7.10 Prociency Exam . . . . . . . . . . . . . . . . . . . . . . . . . . . . . . . . . . . . . . . . . . . . . . . . . . . . . . . . . . . . . . . . . . . . . . . . 464

Solutions . . . . . . . . . . . . . . . . . . . . . . . . . . . . . . . . . . . . . . . . . . . . . . . . . . . . . . . . . . . . . . . . . . . . . . . . . . . . . . . . . . . . . . . 466

8 Techniques of Estimation

8.1 Objectives . . . . . . . . . . . . . . . . . . . . . . . . . . . . . . . . . . . . . . . . . . . . . . . . . . . . . . . . . . . . . . . . . . . . .. . . . . . . . . . . . 477

8.2 Estimation by Rounding . . . . . . . . . . . . . . . . . . . . . . . . . . . . . . . . . . . . . . . . . . . . . . . . . . . . . . .. . . . . . . . . . . . 477

8.3 Estimation by Clustering . . . . . . . . . . . . . . . . . . . . . . . . . . . . . . . . . . . . . . . . . . . . . . . . . . . . . . . . . . . . . . . . . . 484

8.4 Mental Arithmetic-Using the Distributive Property . . . . . . . . . . . . . . . . . . . . . . . . . . . . . . . . . . . . . . . . 487

8.5 Estimation by Rounding Fractions . . . . . . . . . . . . . . . . . . . . . . . . . . . . . . . . . . . . . . . . . . . . .. . . . . . . . . . . . 493

8.6 Summary of Key Concepts . . . . . . . . . . . . . . . . . . . . . . . . . . . . . . . . . . . . . . . . . . . . . . . . . . . . . . . . . . . . . . . . 496

8.7 Exercise Supplement . . . . . . . . . . . . . . . . . . . . . . . . . . . . . . . . . . . . . . . . . . . . . . . . . . . . . . . . . . . . . . . . . . . . . . 496

8.8 Prociency Exam . . . . . . . . . . . . . . . . . . . . . . . . . . . . . . . . . . . . . . . . . . . . . . . . . . . . . . . . . . . . . .. . . . . . . . . . . . 502

Available for free at Connexions <http://cnx.org/content/col10615/1.4>

v

Solutions . . . . . . . . . . . . . . . . . . . . . . . . . . . . . . . . . . . . . . . . . . . . . . . . . . . . . . . . . . . . . . . . . . . . . . . . . . . . . . . . . . . . . . . 505

9 Measurement and Geometry

9.1 Objectives . . . . . . . . . . . . . . . . . . . . . . . . . . . . . . . . . . . . . . . . . . . . . . . . . . . . . . . . . . . . . . . . . . . . .. . . . . . . . . . . . 513

9.2 Measurement and the United States System . . . . . . . . . . . . . . . . . . . . . . . . . . . . . . . . . . . . . . . . . . . . . . . 514

9.3 The Metric System of Measurement . . . . . . . . . . . . . . . . . . . . . . . . . . . . . . . . . . . . . . . . . . . . . . . . . . . . . . . 519

9.4 Simplication of Denominate Numbers . . . . . . . . . . . . . . . . . . . . . . . . . . . . . . . . . . . . . . . . . . . . . . . . . . . . 524

9.5 Perimeter and Circumference of Geometric Figures . . . . . . . . . . . . . . . . . . . . . . . . . . . . . . . . . . . . . . . . 532

9.6 Area and Volume of Geometric Figures and Objects . . . . . . . . . . . . . . . . . . . . . . . . . . . . . . . . . . . . . . . 542

9.7 Summary of Key Concepts . . . . . . . . . . . . . . . . . . . . . . . . . . . . . . . . . . . . . . . . . . . . . . . . . . . . . . . . . . . . . . . . 557

9.8 Exercise Supplement . . . . . . . . . . . . . . . . . . . . . . . . . . . . . . . . . . . . . . . . . . . . . . . . . . . . . . . . . . . . . . . . . . . . . . 559

9.9 Prociency Exam . . . . . . . . . . . . . . . . . . . . . . . . . . . . . . . . . . . . . . . . . . . . . . . . . . . . . . . . . . . . . .. . . . . . . . . . . . 564

Solutions . . . . . . . . . . . . . . . . . . . . . . . . . . . . . . . . . . . . . . . . . . . . . . . . . . . . . . . . . . . . . . . . . . . . . . . . . . . . . . . . . . . . . . . 569

10 Signed Numbers

10.1 Objectives . . . . . . . . . . . . . . . . . . . . . . . . . . . . . . . . . . . . . . . . . . . . . . . . . . . . . . . . . . . . . . . . . . . . . . . . . . . . . . . 577

10.2 Variables, Constants, and Real Numbers . . . . . . . . . . . . . . . . . . . . . . . . . . . . . . . . . . . . . . . . . . . . . . . . . 578

10.3 Signed Numbers . . . . . . . . . . . . . . . . . . . . . . . . . . . . . . . . . . . . . . . . . . . . . . . . . . . . . . . . . . . . . . . . . . . . . . . . . 584

10.4 Absolute Value . . . . . . . . . . . . . . . . . . . . . . . . . . . . . . . . . . . . . . . . . . . . . . . . . . . . . . . . . . . . . . .. . . . . . . . . . . . 588

10.5 Addition of Signed Numbers . . . . . . . . . . . . . . . . . . . . . . . . . . . . . . . . . . . . . . . . . . . . . . . . . . . . . . . . . . . . . 592

10.6 Subtraction of Signed Numbers . . . . . . . . . . . . . . . . . . . . . . . . . . . . . . . . . . . . . . . . . . . . . . .. . . . . . . . . . . . 599

10.7 Multiplication and Division of Signed Numbers . . . . . . . . . . . . . . . . . . . . . . . . . . . . . . .. . . . . . . . . . . . 604

10.8 Summary of Key Concepts . . . . . . . . . . . . . . . . . . . . . . . . . . . . . . . . . . . . . . . . . . . . . . . . . . . . . . . . . . . . . . . 613

10.9 Exercise Supplement . . . . . . . . . . . . . . . . . . . . . . . . . . . . . . . . . . . . . . . . . . . . . . . . . . . . . . . . . . . . . . . . . . . . . 614

10.10 Prociency Exam . . . . . . . . . . . . . . . . . . . . . . . . . . . . . . . . . . . . . . . . . . . . . . . . . . . . . . . . . . . . . . . . . . . . . . . 618

Solutions . . . . . . . . . . . . . . . . . . . . . . . . . . . . . . . . . . . . . . . . . . . . . . . . . . . . . . . . . . . . . . . . . . . . . . . . . . . . . . . . . . . . . . . 620

11 Algebraic Expressions and Equations

11.1 Objectives . . . . . . . . . . . . . . . . . . . . . . . . . . . . . . . . . . . . . . . . . . . . . . . . . . . . . . . . . . . . . . . . . . . . . . . . . . . . . . . 631

11.2 Algebraic Expressions . . . . . . . . . . . . . . . . . . . . . . . . . . . . . . . . . . . . . . . . . . . . . . . . . . . . . . . . . . . . . . . . . . . . 632

11.3 Combining Like Terms Using Addition and Subtraction . . . . . . . . . . . . . . . . . . . . . . . . . . . . . . . . . . 638

11.4 Solving Equations of the Form x+a=b and x-a=b . . . . . . . . . . . . . . . . . . . . . . . . . . . . . . . . . . . . . . . . 641

11.5 Solving Equations of the Form ax=b and x/a=b . . . . . . . . . . . . . . . . . . . . . . . . . . . . . . . . . . . . . . . . . 649

11.6 Applications I: Translating Words to Mathematical Symbols . . . . . . . . . . . . . . . . . .. . . . . . . . . . . . 657

11.7 Applications II: Solving Problems . . . . . . . . . . . . . . . . . . . . . . . . . . . . . . . . . . . . . . . . . . . . . . . . . . . . . . . . 662

11.8 Summary of Key Concepts . . . . . . . . . . . . . . . . . . . . . . . . . . . . . . . . . . . . . . . . . . . . . . . . . . . . . . . . . . . . . . . 672

11.9 Exercise Supplement . . . . . . . . . . . . . . . . . . . . . . . . . . . . . . . . . . . . . . . . . . . . . . . . . . . . . . . . . . . . . . . . . . . . . 674

11.10 Prociency Exam . . . . . . . . . . . . . . . . . . . . . . . . . . . . . . . . . . . . . . . . . . . . . . . . . . . . . . . . . . . . . . . . . . . . . . . 680

Solutions . . . . . . . . . . . . . . . . . . . . . . . . . . . . . . . . . . . . . . . . . . . . . . . . . . . . . . . . . . . . . . . . . . . . . . . . . . . . . . . . . . . . . . . 682

Index . . . . . . . . . . . . . . . . . . . . . . . . . . . . . . . . . . . . . . . . . . . . . . . . . . . . . . . . . . . . . . . . . . . . . . . . . . . . . . . . . . . . . . . . . . . . . . . 694

Attributions . . . . . . . . . . . . . . . . . . . . . . . . . . . . . . . . . . . . . . . . . . . . . . . . . . . . . . . . . . . . . . . . . . . . . . . . . . . . . . . . . . . . . . . .699

Available for free at Connexions <http://cnx.org/content/col10615/1.4>

vi

Available for free at Connexions <http://cnx.org/content/col10615/1.4>

Preface1

To the next generation of explorers: Kristi, BreAnne, Lindsey, Randi, Piper, Meghan, Wyatt, Lara, Mason, and Sheanna.

Fundamentals of Mathematics is a work text that covers the traditional topics studied in a modern prealgebra course, as well as the topics of estimation, elementary analytic geometry, and introductory algebra. It is intended for students who

1. have had a previous course in prealgebra,

2. wish to meet the prerequisite of a higher level course such as elementary algebra, and 3. need to review fundamental mathematical concepts and techniques.

This text will help the student develop the insight and intuition necessary to master arithmetic techniques and manipulative skills. It was written with the following main objectives: 1. to provide the student with an understandable and usable source of information, 2. to provide the student with the maximum opportunity to see that arithmetic concepts and techniques are logically based,

3. to instill in the student the understanding and intuitive skills necessary to know how and when to use particular arithmetic concepts in subsequent material, courses, and nonclassroom situations, and 4. to give the student the ability to correctly interpret arithmetically obtained results.

We have tried to meet these objectives by presenting material dynamically, much the way an instructor might present the material visually in a classroom. (See the development of the concept of addition and subtraction of fractions in Section 5.3, for example.) Intuition and understanding are some of the keys to creative thinking; we believe that the material presented in this text will help the student realize that mathematics is a creative subject.

This text can be used in standard lecture or self-paced classes. To help meet our objectives and to make the study of prealgebra a pleasant and rewarding experience, Fundamentals of Mathematics is organized as follows.

Pedagogical Features

The work text format gives the student space to practice mathematical skills with ready reference to sample problems. The chapters are divided into sections, and each section is a complete treatment of a particular topic, which includes the following features:

• Section Overview

• Sample Sets

• Practice Sets

1This content is available online at <http://cnx.org/content/m18884/1.4/>.

Available for free at Connexions <http://cnx.org/content/col10615/1.4> 1

2

• Section Exercises

• Exercises for Review

• Answers to Practice Sets

The chapters begin with Objectives and end with a Summary of Key Concepts, an Exercise Supplement, and a Prociency Exam.

Objectives

Each chapter begins with a set of objectives identifying the material to be covered. Each section begins with an overview that repeats the objectives for that particular section. Sections are divided into subsections that correspond to the section objectives, which makes for easier reading.

Sample Sets

Fundamentals of Mathematics contains examples that are set o in boxes for easy reference. The examples are referred to as Sample Sets for two reasons:

1. They serve as a representation to be imitated, which we believe will foster understanding of mathematical concepts and provide experience with mathematical techniques.

2. Sample Sets also serve as a preliminary representation of problem-solving techniques that may be used to solve more general and more complicated problems.

The examples have been carefully chosen to illustrate and develop concepts and techniques in the most instructive, easily remembered way. Concepts and techniques preceding the examples are introduced at a level below that normally used in similar texts and are thoroughly explained, assuming little previous knowledge.

Practice Sets

A parallel Practice Set follows each Sample Set, which reinforces the concepts just learned. There is adequate space for the student to work each problem directly on the page.

Answers to Practice Sets

The Answers to Practice Sets are given at the end of each section and can be easily located by referring to the page number, which appears after the last Practice Set in each section.

Section Exercises

The exercises at the end of each section are graded in terms of diculty, although they are not grouped into categories. There is an ample number of problems, and after working through the exercises, the student will be capable of solving a variety of challenging problems.

The problems are paired so that the odd-numbered problems are equivalent in kind and diculty to the even-numbered problems. Answers to the odd-numbered problems are provided at the back of the book.

Exercises for Review

This section consists of ve problems that form a cumulative review of the material covered in the preceding sections of the text and is not limited to material in that chapter. The exercises are keyed by section for easy reference. Since these exercises are intended for review only, no work space is provided.

Summary of Key Concepts

A summary of the important ideas and formulas used throughout the chapter is included at the end of each chapter. More than just a list of terms, the summary is a valuable tool that reinforces concepts in preparation for the Prociency Exam at the end of the chapter, as well as future exams. The summary keys each item to the section of the text where it is discussed.

Available for free at Connexions <http://cnx.org/content/col10615/1.4>

3

Exercise Supplement

In addition to numerous section exercises, each chapter includes approximately 100 supplemental problems, which are referenced by section. Answers to the odd-numbered problems are included in the back of the book.

Prociency Exam

Each chapter ends with a Prociency Exam that can serve as a chapter review or evaluation. The Prociency Exam is keyed to sections, which enables the student to refer back to the text for assistance. Answers to all the problems are included in the Answer Section at the end of the book.

Content

The writing style used in Fundamentals of Mathematics is informal and friendly, oering a straightforward approach to prealgebra mathematics. We have made a deliberate eort not to write another text that mini-mizes the use of words because we believe that students can best study arithmetic concepts and understand arithmetic techniques by using words and symbols rather than symbols alone. It has been our experience that students at the prealgebra level are not nearly experienced enough with mathematics to understand symbolic explanations alone; they need literal explanations to guide them through the symbols.

We have taken great care to present concepts and techniques so they are understandable and easily remembered. After concepts have been developed, students are warned about common pitfalls. We have tried to make the text an information source accessible to prealgebra students.

Addition and Subtraction of Whole Numbers

This chapter includes the study of whole numbers, including a discussion of the Hindu-Arabic numeration and the base ten number systems. Rounding whole numbers is also presented, as are the commutative and associative properties of addition.

Multiplication and Division of Whole Numbers

The operations of multiplication and division of whole numbers are explained in this chapter. Multiplication is described as repeated addition. Viewing multiplication in this way may provide students with a visualization of the meaning of algebraic terms such as 8x when they start learning algebra. The chapter also includes the commutative and associative properties of multiplication.

Exponents, Roots, and Factorizations of Whole Numbers

The concept and meaning of the word root is introduced in this chapter. A method of reading root notation and a method of determining some common roots, both mentally and by calculator, is then presented. We also present grouping symbols and the order of operations, prime factorization of whole numbers, and the greatest common factor and least common multiple of a collection of whole numbers.

Introduction to Fractions and Multiplication and Division of Fractions

We recognize that fractions constitute one of the foundations of problem solving. We have, therefore, given a detailed treatment of the operations of multiplication and division of fractions and the logic behind these operations. We believe that the logical treatment and many practice exercises will help students retain the information presented in this chapter and enable them to use it as a foundation for the study of rational expressions in an algebra course.

Available for free at Connexions <http://cnx.org/content/col10615/1.4>

4

Addition and Subtraction of Fractions, Comparing Fractions, and Complex Fractions

A detailed treatment of the operations of addition and subtraction of fractions and the logic behind these operations is given in this chapter. Again, we believe that the logical treatment and many practice exercises will help students retain the information, thus enabling them to use it in the study of rational expressions in an algebra course. We have tried to make explanations dynamic. A method for comparing fractions is introduced, which gives the student another way of understanding the relationship between the words denominator and denomination. This method serves to show the student that it is sometimes possible to compare two dierent types of quantities. We also study a method of simplifying complex fractions and of combining operations with fractions.

Decimals

The student is introduced to decimals in terms of the base ten number system, fractions, and digits occurring to the right of the units position. A method of converting a fraction to a decimal is discussed. The logic behind the standard methods of operating on decimals is presented and many examples of how to apply the methods are given. The word of as related to the operation of multiplication is discussed. Nonterminating divisions are examined, as are combinations of operations with decimals and fractions.

Ratios and Rates

We begin by dening and distinguishing the terms ratio and rate. The meaning of proportion and some applications of proportion problems are described. Proportion problems are solved using the “Five-Step Method.” We hope that by using this method the student will discover the value of introducing a variable as a rst step in problem solving and the power of organization. The chapter concludes with discussions of percent, fractions of one percent, and some applications of percent.

Techniques of Estimation

One of the most powerful problem-solving tools is a knowledge of estimation techniques. We feel that estimation is so important that we devote an entire chapter to its study. We examine three estimation techniques: estimation by rounding, estimation by clustering, and estimation by rounding fractions. We also include a section on the distributive property, an important algebraic property.

Measurement and Geometry

This chapter presents some of the techniques of measurement in both the United States system and the metric system. Conversion from one unit to another (in a system) is examined in terms of unit fractions. A discussion of the simplication of denominate numbers is also included. This discussion helps the student understand more clearly the association between pure numbers and dimensions. The chapter concludes with a study of perimeter and circumference of geometric gures and area and volume of geometric gures and objects.

Signed Numbers

A look at algebraic concepts and techniques is begun in this chapter. Basic to the study of algebra is a working knowledge of signed numbers. Denitions of variables, constants, and real numbers are introduced. We then distinguish between positive and negative numbers, learn how to read signed numbers, and examine the origin and use of the double-negative property of real numbers. The concept of absolute value is presented both geometrically (using the number line) and algebraically. The algebraic denition is followed by an interpretation of its meaning and several detailed examples of its use. Addition, subtraction, multiplication, and division of signed numbers are presented rst using the number line, then with absolute value.

Available for free at Connexions <http://cnx.org/content/col10615/1.4>

5

Algebraic Expressions and Equations

The student is introduced to some elementary algebraic concepts and techniques in this nal chapter. Algebraic expressions and the process of combining like terms are discussed in Section 11.2 and Section 11.3.

The method of combining like terms in an algebraic expression is explained by using the interpretation of multiplication as a description of repeated addition (as in Section 2.1).

Available for free at Connexions <http://cnx.org/content/col10615/1.4>

6

Available for free at Connexions <http://cnx.org/content/col10615/1.4>

Acknowledgements2

Many extraordinarily talented people are responsible for helping to create this text. We wish to acknowledge the eorts and skill of the following mathematicians. Their contributions have been invaluable.

Barbara Conway, Berkshire Community College

Bill Hajdukiewicz, Miami-Dade Community College

Virginia Hamilton, Shawnee State University

David Hares, El Centro College

Norman Lee, Ball State University

Ginger Y. Manchester, Hinds Junior College

John R. Martin, Tarrant County Junior College

Shelba Mormon, Northlake College

Lou Ann Pate, Pima Community College

Gus Pekara, Oklahoma City Community College

David Price, Tarrant County Junior College

David Schultz, Virginia Western Community College

Sue S. Watkins, Lorain County Community College

Elizabeth M. Wayt, Tennessee State University

Prentice E. Whitlock, Jersey City State College

Thomas E. Williamson, Montclair State College

Special thanks to the following individuals for their careful accuracy reviews of manuscript, galleys, and page proofs: Steve Blasberg, West Valley College; Wade Ellis, Sr., University of Michigan; John R. Martin, Tarrant County Junior College; and Jane Ellis. We would also like to thank Amy Miller and Guy Sanders, Branham High School.

Our sincere thanks to Debbie Wiedemann for her encouragement, suggestions concerning psychobiological examples, proofreading much of the manuscript, and typing many of the section exercises; Sandi Wiedemann for collating the annotated reviews, counting the examples and exercises, and untiring use of “white-out”; and Jane Ellis for solving and typing all of the exercise solutions.

We thank the following people for their excellent work on the various ancillary items that accompany Fundamentals of Mathematics: Steve Blasberg, West Valley College; Wade Ellis, Sr., University of Michigan; and Jane Ellis ( Instructor’s Manual); John R. Martin, Tarrant County Junior College (Student Solutions Manual and Study Guide); Virginia Hamilton, Shawnee State University (Computerized Test Bank); Pa-tricia Morgan, San Diego State University (Prepared Tests); and George W. Bergeman, Northern Virginia Community College (Maxis Interactive Software).

We also thank the talented people at Saunders College Publishing whose eorts made this text run smoothly and less painfully than we had imagined. Our particular thanks to Bob Stern, Mathematics Editor, Ellen Newman, Developmental Editor, and Janet Nuciforo, Project Editor. Their guidance, suggestions, open 2This content is available online at <http://cnx.org/content/m34775/1.2/>.

Available for free at Connexions <http://cnx.org/content/col10615/1.4> 7

8

minds to our suggestions and concerns, and encouragement have been extraordinarily helpful. Although there were times we thought we might be permanently damaged from rereading and rewriting, their eorts have improved this text immensely. It is a pleasure to work with such high-quality professionals.

Denny Burzynski

Wade Ellis, Jr.

San Jose, California

December 1988

I would like to thank Doug Campbell, Ed Lodi, and Guy Sanders for listening to my frustrations and encouraging me on. Thanks also go to my cousin, David Raety, who long ago in Sequoia National Forest told me what a dierential equation is.

Particular thanks go to each of my colleagues at West Valley College. Our everyday conversations regarding mathematics instruction have been of the utmost importance to the development of this text and to my teaching career.

D.B.

Available for free at Connexions <http://cnx.org/content/col10615/1.4>

Chapter 1

Addition and Subtraction of Whole

Numbers

1.1 Objectives1

After completing this chapter, you should

Whole Numbers (Section 1.2)

• know the dierence between numbers and numerals

• know why our number system is called the Hindu-Arabic numeration system

• understand the base ten positional number system

• be able to identify and graph whole numbers

Reading and Writing Whole Numbers (Section 1.3)

• be able to read and write a whole number

Rounding Whole Numbers (Section 1.4)

• understand that rounding is a method of approximation

• be able to round a whole number to a specied position

Addition of Whole Numbers (Section 1.5)

• understand the addition process

• be able to add whole numbers

• be able to use the calculator to add one whole number to another

Subtraction of Whole Numbers (Section 1.6)

• understand the subtraction process

• be able to subtract whole numbers

• be able to use a calculator to subtract one whole number from another whole number Properties of Addition (Section 1.7)

• understand the commutative and associative properties of addition

• understand why 0 is the additive identity

1This content is available online at <http://cnx.org/content/m18885/1.4/>.

Available for free at Connexions <http://cnx.org/content/col10615/1.4> 9

Image 1

10

CHAPTER 1. ADDITION AND SUBTRACTION OF WHOLE NUMBERS

1.2 Whole Numbers2

1.2.1 Section Overview

• Numbers and Numerals

• The Hindu-Arabic Numeration System

• The Base Ten Positional Number System

• Whole Numbers

• Graphing Whole Numbers

1.2.2 Numbers and Numerals

We begin our study of introductory mathematics by examining its most basic building block, the number.

Number

A number is a concept. It exists only in the mind.

The earliest concept of a number was a thought that allowed people to mentally picture the size of some collection of objects. To write down the number being conceptualized, a numeral is used.

Numeral

A numeral is a symbol that represents a number.

In common usage today we do not distinguish between a number and a numeral. In our study of introductory mathematics, we will follow this common usage.

1.2.2.1 Sample Set A

The following are numerals. In each case, the rst represents the number four, the second represents the number one hundred twenty-three, and the third, the number one thousand ve. These numbers are represented in dierent ways.

• Hindu-Arabic numerals

4, 123, 1005

• Roman numerals

IV, CXXIII, MV

• Egyptian numerals

1.2.2.2 Practice Set A

Exercise 1.2.1

(Solution on p. 76.)

Do the phrases “four,” “one hundred twenty-three,” and “one thousand ve” qualify as numerals?

Yes or no?

2This content is available online at <http://cnx.org/content/m34795/1.5/>.

Available for free at Connexions <http://cnx.org/content/col10615/1.4>

Image 2

Image 3

11

1.2.3 The Hindu-Arabic Numeration System

Hindu-Arabic Numeration System

Our society uses the Hindu-Arabic numeration system. This system of numeration began shortly before the third century when the Hindus invented the numerals

0 1 2 3 4 5 6 7 8 9

Leonardo Fibonacci

About a thousand years later, in the thirteenth century, a mathematician named Leonardo Fibonacci of Pisa introduced the system into Europe. It was then popularized by the Arabs. Thus, the name, Hindu-Arabic numeration system.

1.2.4 The Base Ten Positional Number System

Digits

The Hindu-Arabic numerals 0 1 2 3 4 5 6 7 8 9 are called digits. We can form any number in the number system by selecting one or more digits and placing them in certain positions. Each position has a particular value. The Hindu mathematician who devised the system about A.D. 500 stated that “from place to place each is ten times the preceding.”

Base Ten Positional Systems

It is for this reason that our number system is called a positional number system with base ten.

Commas

When numbers are composed of more than three digits, commas are sometimes used to separate the digits into groups of three.

Periods

These groups of three are called periods and they greatly simplify reading numbers.

In the Hindu-Arabic numeration system, a period has a value assigned to each or its three positions, and the values are the same for each period. The position values are

Thus, each period contains a position for the values of one, ten, and hundred. Notice that, in looking from right to left, the value of each position is ten times the preceding. Each period has a particular name.

As we continue from right to left, there are more periods. The ve periods listed above are the most common, and in our study of introductory mathematics, they are sucient.

The following diagram illustrates our positional number system to trillions. (There are, to be sure, other periods.)

Available for free at Connexions <http://cnx.org/content/col10615/1.4>

Image 4

12

CHAPTER 1. ADDITION AND SUBTRACTION OF WHOLE NUMBERS

In our positional number system, the value of a digit is determined by its position in the number.

1.2.4.1 Sample Set B

Example 1.1

Find the value of 6 in the number 7,261.

Since 6 is in the tens position of the units period, its value is 6 tens.

6 tens = 60

Example 1.2

Find the value of 9 in the number 86,932,106,005.

Since 9 is in the hundreds position of the millions period, its value is 9 hundred millions.

9 hundred millions = 9 hundred million

Example 1.3

Find the value of 2 in the number 102,001.

Since 2 is in the ones position of the thousands period, its value is 2 one thousands.

2 one thousands = 2 thousand

1.2.4.2 Practice Set B

Exercise 1.2.2

(Solution on p. 76.)

Find the value of 5 in the number 65,000.

Exercise 1.2.3

(Solution on p. 76.)

Find the value of 4 in the number 439,997,007,010.

Exercise 1.2.4

(Solution on p. 76.)

Find the value of 0 in the number 108.

1.2.5 Whole Numbers

Whole Numbers

Numbers that are formed using only the digits

0 1 2 3 4 5 6 7 8 9

are called whole numbers. They are

0, 1, 2, 3, 4, 5, 6, 7, 8, 9, 10, 11, 12, 13, 14, 15, . . .

The three dots at the end mean “and so on in this same pattern.”

Available for free at Connexions <http://cnx.org/content/col10615/1.4>

Image 5

Image 6

Image 7

Image 8

Image 9

13

1.2.6 Graphing Whole Numbers

Number Line

Whole numbers may be visualized by constructing a number line. To construct a number line, we simply draw a straight line and choose any point on the line and label it 0.

Origin

This point is called the origin. We then choose some convenient length, and moving to the right, mark o consecutive intervals (parts) along the line starting at 0. We label each new interval endpoint with the next whole number.

Graphing

We can visually display a whole number by drawing a closed circle at the point labeled with that whole number. Another phrase for visually displaying a whole number is graphing the whole number. The word graph means to “visually display.”

1.2.6.1 Sample Set C

Example 1.4

Graph the following whole numbers: 3, 5, 9.

Example 1.5

Specify the whole numbers that are graphed on the following number line. The break in the number line indicates that we are aware of the whole numbers between 0 and 106, and 107 and 872, but we are not listing them due to space limitations.

The numbers that have been graphed are

0, 106, 873, 874

1.2.6.2 Practice Set C

Exercise 1.2.5

(Solution on p. 76.)

Graph the following whole numbers: 46, 47, 48, 325, 327.

Exercise 1.2.6

(Solution on p. 76.)

Specify the whole numbers that are graphed on the following number line.

A line is composed of an endless number of points. Notice that we have labeled only some of them. As we proceed, we will discover new types of numbers and determine their location on the number line.

Available for free at Connexions <http://cnx.org/content/col10615/1.4>

14

CHAPTER 1. ADDITION AND SUBTRACTION OF WHOLE NUMBERS

1.2.7 Exercises

Exercise 1.2.7

(Solution on p. 76.)

What is a number?

Exercise 1.2.8

What is a numeral?

Exercise 1.2.9

(Solution on p. 76.)

Does the word “eleven” qualify as a numeral?

Exercise 1.2.10

How many dierent digits are there?

Exercise 1.2.11

(Solution on p. 76.)

Our number system, the Hindu-Arabic number system, is a

number system with

base

.

Exercise 1.2.12

Numbers composed of more than three digits are sometimes separated into groups of three by commas. These groups of three are called

.

Exercise 1.2.13

(Solution on p. 76.)

In our number system, each period has three values assigned to it. These values are the same for each period. From right to left, what are they?

Exercise 1.2.14

Each period has its own particular name. From right to left, what are the names of the rst four?

Exercise 1.2.15

(Solution on p. 76.)

In the number 841, how many tens are there?

Exercise 1.2.16

In the number 3,392, how many ones are there?

Exercise 1.2.17

(Solution on p. 76.)

In the number 10,046, how many thousands are there?

Exercise 1.2.18

In the number 779,844,205, how many ten millions are there?

Exercise 1.2.19

(Solution on p. 76.)

In the number 65,021, how many hundred thousands are there?

For following problems, give the value of the indicated digit in the given number.

Exercise 1.2.20

5 in 599

Exercise 1.2.21

(Solution on p. 76.)

1 in 310,406

Exercise 1.2.22

9 in 29,827

Exercise 1.2.23

(Solution on p. 76.)

6 in 52,561,001,100

Exercise 1.2.24

Write a two-digit number that has an eight in the tens position.

Exercise 1.2.25

(Solution on p. 76.)

Write a four-digit number that has a one in the thousands position and a zero in the ones position.

Exercise 1.2.26

How many two-digit whole numbers are there?

Available for free at Connexions <http://cnx.org/content/col10615/1.4>

Image 10

Image 11

Image 12

15

Exercise 1.2.27

(Solution on p. 76.)

How many three-digit whole numbers are there?

Exercise 1.2.28

How many four-digit whole numbers are there?

Exercise 1.2.29

(Solution on p. 76.)

Is there a smallest whole number? If so, what is it?

Exercise 1.2.30

Is there a largest whole number? If so, what is it?

Exercise 1.2.31

(Solution on p. 76.)

Another term for “visually displaying” is

.

Exercise 1.2.32

The whole numbers can be visually displayed on a

.

Exercise 1.2.33

(Solution on p. 76.)

Graph (visually display) the following whole numbers on the number line below: 0, 1, 31, 34.

Exercise 1.2.34

Construct a number line in the space provided below and graph (visually display) the following whole numbers: 84, 85, 901, 1006, 1007.

Exercise 1.2.35

(Solution on p. 76.)

Specify, if any, the whole numbers that are graphed on the following number line.

Exercise 1.2.36

Specify, if any, the whole numbers that are graphed on the following number line.

1.3 Reading and Writing Whole Numbers3

1.3.1 Section Overview

• Reading Whole Numbers

• Writing Whole Numbers

Because our number system is a positional number system, reading and writing whole numbers is quite simple.

3This content is available online at <http://cnx.org/content/m34778/1.5/>.

Available for free at Connexions <http://cnx.org/content/col10615/1.4>

Image 13

Image 14

Image 15

Image 16

Image 17

16

CHAPTER 1. ADDITION AND SUBTRACTION OF WHOLE NUMBERS

1.3.2 Reading Whole Numbers

To convert a number that is formed by digits into a verbal phrase, use the following method: 1. Beginning at the right and working right to left, separate the number into distinct periods by inserting commas every three digits.

2. Beginning at the left, read each period individually, saying the period name.

1.3.2.1 Sample Set A

Write the following numbers as words.

Example 1.6

Read 42958.

1. Beginning at the right, we can separate this number into distinct periods by inserting a comma between the 2 and 9.

42,958

2. Beginning at the left, we read each period individually:

Forty-two thousand, nine hundred fty-eight.

Example 1.7

Read 307991343.

1. Beginning at the right, we can separate this number into distinct periods by placing commas between the 1 and 3 and the 7 and 9.

307,991,343

2. Beginning at the left, we read each period individually.

Three hundred seven million, nine hundred ninety-one thousand, three hundred forty-three.

Example 1.8

Read 36000000000001.

1. Beginning at the right, we can separate this number into distinct periods by placing commas.

36,000,000,001

Available for free at Connexions <http://cnx.org/content/col10615/1.4>

Image 18

17

2. Beginning at the left, we read each period individually.

Thirty-six trillion, one.

1.3.2.2 Practice Set A

Write each number in words.

Exercise 1.3.1

(Solution on p. 76.)

12,542

Exercise 1.3.2

(Solution on p. 76.)

101,074,003

Exercise 1.3.3

(Solution on p. 76.)

1,000,008

1.3.3 Writing Whole Numbers

To express a number in digits that is expressed in words, use the following method: 1. Notice rst that a number expressed as a verbal phrase will have its periods set o by commas.

2. Starting at the beginning of the phrase, write each period of numbers individually.

3. Using commas to separate periods, combine the periods to form one number.

1.3.3.1 Sample Set B

Write each number using digits.

Example 1.9

Seven thousand, ninety-two.

Using the comma as a period separator, we have

Available for free at Connexions <http://cnx.org/content/col10615/1.4>

Image 19

Image 20

Image 21

18

CHAPTER 1. ADDITION AND SUBTRACTION OF WHOLE NUMBERS

7,092

Example 1.10

Fifty billion, one million, two hundred thousand, fourteen.

Using the commas as period separators, we have

50,001,200,014

Example 1.11

Ten million, ve hundred twelve.

The comma sets o the periods. We notice that there is no thousands period. We’ll have to insert this ourselves.

10,000,512

1.3.3.2 Practice Set B

Express each number using digits.

Exercise 1.3.4

(Solution on p. 77.)

One hundred three thousand, twenty-ve.

Exercise 1.3.5

(Solution on p. 77.)

Six million, forty thousand, seven.

Exercise 1.3.6

(Solution on p. 77.)

Twenty trillion, three billion, eighty million, one hundred nine thousand, four hundred two.

Exercise 1.3.7

(Solution on p. 77.)

Eighty billion, thirty-ve.

1.3.4 Exercises

For the following problems, write all numbers in words.

Exercise 1.3.8

(Solution on p. 77.)

912

Exercise 1.3.9

84

Available for free at Connexions <http://cnx.org/content/col10615/1.4>

19

Exercise 1.3.10

(Solution on p. 77.)

1491

Exercise 1.3.11

8601

Exercise 1.3.12

(Solution on p. 77.)

35,223

Exercise 1.3.13

71,006

Exercise 1.3.14

(Solution on p. 77.)

437,105

Exercise 1.3.15

201,040

Exercise 1.3.16

(Solution on p. 77.)

8,001,001

Exercise 1.3.17

16,000,053

Exercise 1.3.18

(Solution on p. 77.)

770,311,101

Exercise 1.3.19

83,000,000,007

Exercise 1.3.20

(Solution on p. 77.)

106,100,001,010

Exercise 1.3.21

3,333,444,777

Exercise 1.3.22

(Solution on p. 77.)

800,000,800,000

Exercise 1.3.23

A particular community college has 12,471 students enrolled.

Exercise 1.3.24

(Solution on p. 77.)

A person who watches 4 hours of television a day spends 1460 hours a year watching T.V.

Exercise 1.3.25

Astronomers believe that the age of the earth is about 4,500,000,000 years.

Exercise 1.3.26

(Solution on p. 77.)

Astronomers believe that the age of the universe is about 20,000,000,000 years.

Exercise 1.3.27

There are 9690 ways to choose four objects from a collection of 20.

Exercise 1.3.28

(Solution on p. 77.)

If a 412 page book has about 52 sentences per page, it will contain about 21,424 sentences.

Exercise 1.3.29

In 1980, in the United States, there was $1,761,000,000,000 invested in life insurance.

Exercise 1.3.30

(Solution on p. 77.)

In 1979, there were 85,000 telephones in Alaska and 2,905,000 telephones in Indiana.

Exercise 1.3.31

In 1975, in the United States, it is estimated that 52,294,000 people drove to work alone.

Exercise 1.3.32

(Solution on p. 77.)

In 1980, there were 217 prisoners under death sentence that were divorced.

Available for free at Connexions <http://cnx.org/content/col10615/1.4>

20

CHAPTER 1. ADDITION AND SUBTRACTION OF WHOLE NUMBERS

Exercise 1.3.33

In 1979, the amount of money spent in the United States for regular-session college education was $50,721,000,000,000.

Exercise 1.3.34

(Solution on p. 77.)

In 1981, there were 1,956,000 students majoring in business in U.S. colleges.

Exercise 1.3.35

In 1980, the average fee for initial and follow up visits to a medical doctors oce was about $34.

Exercise 1.3.36

(Solution on p. 77.)

In 1980, there were approximately 13,100 smugglers of aliens apprehended by the Immigration border patrol.

Exercise 1.3.37

In 1980, the state of West Virginia pumped 2,000,000 barrels of crude oil, whereas Texas pumped 975,000,000 barrels.

Exercise 1.3.38

(Solution on p. 77.)

The 1981 population of Uganda was 12,630,000 people.

Exercise 1.3.39

In 1981, the average monthly salary oered to a person with a Master’s degree in mathematics was $1,685.

For the following problems, write each number using digits.

Exercise 1.3.40

(Solution on p. 77.)

Six hundred eighty-one

Exercise 1.3.41

Four hundred ninety

Exercise 1.3.42

(Solution on p. 77.)

Seven thousand, two hundred one

Exercise 1.3.43

Nineteen thousand, sixty-ve

Exercise 1.3.44

(Solution on p. 77.)

Five hundred twelve thousand, three

Exercise 1.3.45

Two million, one hundred thirty-three thousand, eight hundred fty-nine

Exercise 1.3.46

(Solution on p. 77.)

Thirty-ve million, seven thousand, one hundred one

Exercise 1.3.47

One hundred million, one thousand

Exercise 1.3.48

(Solution on p. 77.)

Sixteen billion, fty-nine thousand, four

Exercise 1.3.49

Nine hundred twenty billion, four hundred seventeen million, twenty-one thousand Exercise 1.3.50

(Solution on p. 78.)

Twenty-three billion

Exercise 1.3.51

Fifteen trillion, four billion, nineteen thousand, three hundred ve

Exercise 1.3.52

(Solution on p. 78.)

One hundred trillion, one

Available for free at Connexions <http://cnx.org/content/col10615/1.4>

Image 22

21

1.3.4.1 Exercises for Review

Exercise 1.3.53

(Section 1.2) How many digits are there?

Exercise 1.3.54

(Solution on p. 78.)

(Section 1.2) In the number 6,641, how many tens are there?

Exercise 1.3.55

(Section 1.2) What is the value of 7 in 44,763?

Exercise 1.3.56

(Solution on p. 78.)

(Section 1.2) Is there a smallest whole number? If so, what is it?

Exercise 1.3.57

(Section 1.2) Write a four-digit number with a 9 in the tens position.

1.4 Rounding Whole Numbers4

1.4.1 Section Overview

• Rounding as an Approximation

• The Method of Rounding Numbers

1.4.2 Rounding as an Approximation

A primary use of whole numbers is to keep count of how many objects there are in a collection. Sometimes we’re only interested in the approximate number of objects in the collection rather than the precise number.

For example, there are approximately 20 symbols in the collection below.

The precise number of symbols in the above collection is 18.

Rounding

We often approximate the number of objects in a collection by mentally seeing the collection as occurring in groups of tens, hundreds, thousands, etc. This process of approximation is called rounding. Rounding is very useful in estimation. We will study estimation in Chapter 8.

When we think of a collection as occurring in groups of tens, we say we’re rounding to the nearest ten. When we think of a collection as occurring in groups of hundreds, we say we’re rounding to the nearest hundred.

This idea of rounding continues through thousands, ten thousands, hundred thousands, millions, etc.

The process of rounding whole numbers is illustrated in the following examples.

Example 1.12

Round 67 to the nearest ten.

4This content is available online at <http://cnx.org/content/m34780/1.3/>.

Available for free at Connexions <http://cnx.org/content/col10615/1.4>

Image 23

Image 24

Image 25

22

CHAPTER 1. ADDITION AND SUBTRACTION OF WHOLE NUMBERS

On the number line, 67 is more than halfway from 60 to 70. The digit immediately to the right of the tens digit, the round-o digit, is the indicator for this.

Thus, 67, rounded to the near-

est ten, is 70.

Example 1.13

Round 4,329 to the nearest hundred.

On the number line, 4,329 is less than halfway from 4,300 to 4,400. The digit to the immediate right of the hundreds digit, the round-o digit, is the indicator.

Thus, 4,329, rounded to the

nearest hundred is 4,300.

Example 1.14

Round 16,500 to the nearest thousand.

On the number line, 16,500 is exactly halfway from 16,000 to 17,000.

By convention, when the number to be rounded is exactly halfway between two numbers, it is rounded to the higher number.

Thus, 16,500, rounded to the nearest thousand, is 17,000.

Example 1.15

A person whose salary is $41,450 per year might tell a friend that she makes $41,000 per year. She has rounded 41,450 to the nearest thousand. The number 41,450 is closer to 41,000 than it is to 42,000.

1.4.3 The Method of Rounding Whole Numbers

From the observations made in the preceding examples, we can use the following method to round a whole number to a particular position.

1. Mark the position of the round-o digit.

2. Note the digit to the immediate right of the round-o digit.

Available for free at Connexions <http://cnx.org/content/col10615/1.4>

Image 26

Image 27

Image 28

23

a. If it is less than 5, replace it and all the digits to its right with zeros. Leave the round-o digit unchanged.

b. If it is 5 or larger, replace it and all the digits to its right with zeros. Increase the round-o digit by 1.

1.4.3.1 Sample Set A

Use the method of rounding whole numbers to solve the following problems.

Example 1.16

Round 3,426 to the nearest ten.

1. We are rounding to the tens position. Mark the digit in the tens position 2. Observe the digit immediately to the right of the tens position. It is 6. Since 6 is greater than 5, we round up by replacing 6 with 0 and adding 1 to the digit in the tens position (the round-o position): 2 + 1 = 3 .

3,430

Thus, 3,426 rounded to the nearest ten is 3,430.

Example 1.17

Round 9,614,018,007 to the nearest ten million.

1. We are rounding to the nearest ten million.

2. Observe the digit immediately to the right of the ten millions position. It is 4. Since 4 is less than 5, we round down by replacing 4 and all the digits to its right with zeros.

9,610,000,000

Thus, 9,614,018,007 rounded to the nearest ten million is 9,610,000,000.

Example 1.18

Round 148,422 to the nearest million.

1. Since we are rounding to the nearest million, we’ll have to imagine a digit in the millions position. We’ll write 148,422 as 0,148,422.

2. The digit immediately to the right is 1. Since 1 is less than 5, we’ll round down by replacing it and all the digits to its right with zeros.

0,000,000

This number is 0.

Available for free at Connexions <http://cnx.org/content/col10615/1.4>

Image 29

24

CHAPTER 1. ADDITION AND SUBTRACTION OF WHOLE NUMBERS

Thus, 148,422 rounded to the nearest million is 0.

Example 1.19

Round 397,000 to the nearest ten thousand.

1. We are rounding to the nearest ten thousand.

2. The digit immediately to the right of the ten thousand position is 7. Since 7 is greater than 5, we round up by replacing 7 and all the digits to its right with zeros and adding 1 to the digit in the ten thousands position. But 9 + 1 = 10 and we must carry the 1 to the next (the hundred thousands) position.

400,000

Thus, 397,000 rounded to the nearest ten thousand is 400,000.

1.4.3.2 Practice Set A

Use the method of rounding whole numbers to solve each problem.

Exercise 1.4.1

(Solution on p. 78.)

Round 3387 to the nearest hundred.

Exercise 1.4.2

(Solution on p. 78.)

Round 26,515 to the nearest thousand.

Exercise 1.4.3

(Solution on p. 78.)

Round 30,852,900 to the nearest million.

Exercise 1.4.4

(Solution on p. 78.)

Round 39 to the nearest hundred.

Exercise 1.4.5

(Solution on p. 78.)

Round 59,600 to the nearest thousand.

1.4.4 Exercises

For the following problems, complete the table by rounding each number to the indicated positions.

Exercise 1.4.6

(Solution on p. 78.)

1,642

hundred thousand ten thousand million

Table 1.1

Exercise 1.4.7

5,221

Available for free at Connexions <http://cnx.org/content/col10615/1.4>

25

hundred thousand ten thousand million

Table 1.2

Exercise 1.4.8

(Solution on p. 78.)

91,803

Hundred thousand ten thousand million

Table 1.3

Exercise 1.4.9

106,007

hundred thousand ten thousand million

Table 1.4

Exercise 1.4.10

(Solution on p. 78.)

208

hundred thousand ten thousand million

Table 1.5

Exercise 1.4.11

199

hundred thousand ten thousand million

Table 1.6

Exercise 1.4.12

(Solution on p. 78.)

863

hundred thousand ten thousand million

Table 1.7

Available for free at Connexions <http://cnx.org/content/col10615/1.4>

26

CHAPTER 1. ADDITION AND SUBTRACTION OF WHOLE NUMBERS

Exercise 1.4.13

794

hundred thousand ten thousand million

Table 1.8

Exercise 1.4.14

(Solution on p. 79.)

925

hundred thousand ten thousand million

Table 1.9

Exercise 1.4.15

909

hundred thousand ten thousand million

Table 1.10

Exercise 1.4.16

(Solution on p. 79.)

981

hundred thousand ten thousand million

Table 1.11

Exercise 1.4.17

965

hundred thousand ten thousand million

Table 1.12

Exercise 1.4.18

(Solution on p. 79.)

551,061,285

hundred thousand ten thousand million

Available for free at Connexions <http://cnx.org/content/col10615/1.4>

27

Table 1.13

Exercise 1.4.19

23,047,991,521

hundred thousand ten thousand million

Table 1.14

Exercise 1.4.20

(Solution on p. 79.)

106,999,413,206

Hundred thousand ten thousand million

Table 1.15

Exercise 1.4.21

5,000,000

hundred thousand ten thousand million

Table 1.16

Exercise 1.4.22

(Solution on p. 79.)

8,006,001

hundred thousand ten thousand million

Table 1.17

Exercise 1.4.23

94,312

hundred thousand ten thousand million

Table 1.18

Exercise 1.4.24

(Solution on p. 79.)

33,486

Available for free at Connexions <http://cnx.org/content/col10615/1.4>

28

CHAPTER 1. ADDITION AND SUBTRACTION OF WHOLE NUMBERS

hundred thousand ten thousand million

Table 1.19

Exercise 1.4.25

560,669

hundred thousand ten thousand million

Table 1.20

Exercise 1.4.26

(Solution on p. 80.)

388,551

hundred thousand ten thousand million

Table 1.21

Exercise 1.4.27

4,752

hundred thousand ten thousand million

Table 1.22

Exercise 1.4.28

(Solution on p. 80.)

8,209

hundred thousand ten thousand million

Table 1.23

Exercise 1.4.29

In 1950, there were 5,796 cases of diphtheria reported in the United States. Round to the nearest hundred.

Exercise 1.4.30

(Solution on p. 80.)

In 1979, 19,309,000 people in the United States received federal food stamps. Round to the nearest ten thousand.

Exercise 1.4.31

In 1980, there were 1,105,000 people between 30 and 34 years old enrolled in school. Round to the nearest million.

Available for free at Connexions <http://cnx.org/content/col10615/1.4>

29

Exercise 1.4.32

(Solution on p. 80.)

In 1980, there were 29,100,000 reports of aggravated assaults in the United States. Round to the nearest million.

For the following problems, round the numbers to the position you think is most reasonable for the situation.

Exercise 1.4.33

In 1980, for a city of one million or more, the average annual salary of police and reghters was $16,096.

Exercise 1.4.34

(Solution on p. 80.)

The average percentage of possible sunshine in San Francisco, California, in June is 73%.

Exercise 1.4.35

In 1980, in the state of Connecticut, $3,777,000,000 in defense contract payroll was awarded.

Exercise 1.4.36

(Solution on p. 80.)

In 1980, the federal government paid $5,463,000,000 to Viet Nam veterans and dependants.

Exercise 1.4.37

In 1980, there were 3,377,000 salespeople employed in the United States.

Exercise 1.4.38

(Solution on p. 80.)

In 1948, in New Hampshire, 231,000 popular votes were cast for the president.

Exercise 1.4.39

In 1970, the world production of cigarettes was 2,688,000,000,000.

Exercise 1.4.40

(Solution on p. 80.)

In 1979, the total number of motor vehicle registrations in Florida was 5,395,000.

Exercise 1.4.41

In 1980, there were 1,302,000 registered nurses the United States.

1.4.4.1 Exercises for Review

Exercise 1.4.42

(Solution on p. 80.)

(Section 1.2) There is a term that describes the visual displaying of a number. What is the term?

Exercise 1.4.43

(Section 1.2) What is the value of 5 in 26,518,206?

Exercise 1.4.44

(Solution on p. 80.)

(Section 1.3) Write 42,109 as you would read it.

Exercise 1.4.45

(Section 1.3) Write “six hundred twelve” using digits.

Exercise 1.4.46

(Solution on p. 80.)

(Section 1.3) Write “four billion eight” using digits.

1.5 Addition of Whole Numbers5

1.5.1 Section Overview

• Addition

• Addition Visualized on the Number Line

• The Addition Process

5This content is available online at <http://cnx.org/content/m34786/1.5/>.

Available for free at Connexions <http://cnx.org/content/col10615/1.4>

Image 30

Image 31

Image 32

30

CHAPTER 1. ADDITION AND SUBTRACTION OF WHOLE NUMBERS

• Addition Involving Carrying

• Calculators

1.5.2 Addition

Suppose we have two collections of objects that we combine together to form a third collection. For example, We are combining a collection of four objects with a collection of three objects to obtain a collection of seven objects.

Addition

The process of combining two or more objects (real or intuitive) to form a third, the total, is called addition.

In addition, the numbers being added are called addends or terms, and the total is called the sum. The plus symbol (+) is used to indicate addition, and the equal symbol (=) is used to represent the word

“equal.” For example, 4 + 3 = 7 means “four added to three equals seven.”

1.5.3 Addition Visualized on the Number Line

Addition is easily visualized on the number line. Let’s visualize the addition of 4 and 3 using the number line.

To nd 4 + 3,

1. Start at 0.

2. Move to the right 4 units. We are now located at 4.

3. From 4, move to the right 3 units. We are now located at 7.

Thus, 4 + 3 = 7.

1.5.4 The Addition Process

We’ll study the process of addition by considering the sum of 25 and 43.

25

means

+43

We write this as 68.

We can suggest the following procedure for adding whole numbers using this example.

Example 1.20: The Process of Adding Whole Numbers

To add whole numbers,

The process:

Available for free at Connexions <http://cnx.org/content/col10615/1.4>

31

1. Write the numbers vertically, placing corresponding positions in the same column.

25

+43

2. Add the digits in each column. Start at the right (in the ones position) and move to the left, placing the sum at the bottom.

25

+43

68

Caution: Confusion and incorrect sums can occur when the numbers are not aligned in columns properly. Avoid writing such additions as

25

+43

25

+43

1.5.4.1 Sample Set A

Example 1.21

Add 276 and 103.

276

6 + 3 = 9 .

+103

7 + 0 = 7 .

379

2 + 1 = 3 .

Example 1.22

Add 1459 and 130

9 + 0 = 9 .

1459

5 + 3 = 8 .

+130

4 + 1 = 5 .

1589

1 + 0 = 1 .

In each of these examples, each individual sum does not exceed 9. We will examine individual sums that exceed 9 in the next section.

1.5.4.2 Practice Set A

Perform each addition. Show the expanded form in problems 1 and 2.

Exercise 1.5.1

(Solution on p. 80.)

Add 63 and 25.

Exercise 1.5.2

(Solution on p. 80.)

Add 4,026 and 1,501.

Exercise 1.5.3

(Solution on p. 80.)

Add 231,045 and 36,121.

Available for free at Connexions <http://cnx.org/content/col10615/1.4>

Image 33

Image 34

Image 35

Image 36

32

CHAPTER 1. ADDITION AND SUBTRACTION OF WHOLE NUMBERS

1.5.5 Addition Involving Carrying

It often happens in addition that the sum of the digits in a column will exceed 9. This happens when we add 18 and 34. We show this in expanded form as follows.

Notice that when we add the 8 ones to the 4 ones we get 12 ones. We then convert the 12 ones to 1 ten and 2 ones. In vertical addition, we show this conversion by carrying the ten to the tens column. We write a 1

at the top of the tens column to indicate the carry. This same example is shown in a shorter form as follows: 8 + 4 = 12 Write 2, carry 1 ten to the top of the next column to the left.

1.5.5.1 Sample Set B

Perform the following additions. Use the process of carrying when needed.

Example 1.23

Add 1875 and 358.

5 + 8 = 13

Write 3, carry 1 ten.

1 + 7 + 5 = 13

Write 3, carry 1 hundred.

1 + 8 + 3 = 12

Write 2, carry 1 thousand.

1 + 1 = 2

The sum is 2233.

Example 1.24

Add 89,208 and 4,946.

Available for free at Connexions <http://cnx.org/content/col10615/1.4>

Image 37

Image 38

Image 39

33

8 + 6 = 14

Write 4, carry 1 ten.

1 + 0 + 4 = 5

Write the 5 (nothing to carry).

2 + 9 = 11

Write 1, carry one thousand.

1 + 9 + 4 = 14

Write 4, carry one ten thousand.

1 + 8 = 9

The sum is 94,154.

Example 1.25

Add 38 and 95.

8 + 5 = 13

Write 3, carry 1 ten.

1 + 3 + 9 = 13

Write 3, carry 1 hundred.

1 + 0 = 1

As you proceed with the addition, it is a good idea to keep in mind what is actually happening.

The sum is 133.

Example 1.26

Find the sum 2648, 1359, and 861.

8 + 9 + 1 = 18

Write 8, carry 1 ten.

1 + 4 + 5 + 6 = 16

Write 6, carry 1 hundred.

1 + 6 + 3 + 8 = 18

Write 8, carry 1 thousand.

1 + 2 + 1 = 4

The sum is 4,868.

Numbers other than 1 can be carried as illustrated in Example 1.27.

Example 1.27

Find the sum of the following numbers.

Available for free at Connexions <http://cnx.org/content/col10615/1.4>

Image 40

Image 41

34

CHAPTER 1. ADDITION AND SUBTRACTION OF WHOLE NUMBERS

6 + 5 + 1 + 7 = 19

Write 9, carry the 1.

1 + 1 + 0 + 5 + 1 = 8

Write 8.

0 + 9 + 9 + 8 = 26

Write 6, carry the 2.

2 + 8 + 9 + 8 + 6 = 33

Write 3, carry the 3.

3 + 7 + 3 + 5 = 18

Write 8, carry the 1.

1 + 8 = 9

Write 9.

The sum is 983,689.

Example 1.28

The number of students enrolled at Riemann College in the years 1984, 1985, 1986, and 1987 was 10,406, 9,289, 10,108, and 11,412, respectively. What was the total number of students enrolled at Riemann College in the years 1985, 1986, and 1987?

We can determine the total number of students enrolled by adding 9,289, 10,108, and 11,412, the number of students enrolled in the years 1985, 1986, and 1987.

The total number of students enrolled at Riemann College in the years 1985, 1986, and 1987 was 30,809.

1.5.5.2 Practice Set B

Perform each addition. For the next three problems, show the expanded form.

Exercise 1.5.4

(Solution on p. 80.)

Add 58 and 29.

Exercise 1.5.5

(Solution on p. 81.)

Add 476 and 85.

Exercise 1.5.6

(Solution on p. 81.)

Add 27 and 88.

Exercise 1.5.7

(Solution on p. 81.)

Add 67,898 and 85,627.

For the next three problems, nd the sums.

Exercise 1.5.8

(Solution on p. 81.)

57

26

84

Available for free at Connexions <http://cnx.org/content/col10615/1.4>

35

Exercise 1.5.9

(Solution on p. 81.)

847

825

796

Exercise 1.5.10

(Solution on p. 81.)

16, 945

8, 472

387, 721

21, 059

629

1.5.6 Calculators

Calculators provide a very simple and quick way to nd sums of whole numbers. For the two problems in Sample Set C, assume the use of a calculator that does not require the use of an ENTER key (such as many Hewlett-Packard calculators).

1.5.6.1 Sample Set C

Use a calculator to nd each sum.

Example 1.29

34 + 21

Display Reads

Type 34 34

Press +

34

Type 21 21

Press =

55

Table 1.24

The sum is 55.

Example 1.30

Available for free at Connexions <http://cnx.org/content/col10615/1.4>

36

CHAPTER 1. ADDITION AND SUBTRACTION OF WHOLE NUMBERS

106 + 85 + 322 + 406 Display Reads

Type 106

106

The calculator keeps a running subtotal

Press +

106

Type 85

85

Press =

191

← 106 + 85

Type 322

322

Press +

513

← 191 + 322

Type 406

406

Press =

919

← 513 + 406

Table 1.25

The sum is 919.

1.5.6.2 Practice Set C

Use a calculator to nd the following sums.

Exercise 1.5.11

(Solution on p. 81.)

62 + 81 + 12

Exercise 1.5.12

(Solution on p. 81.)

9, 261 + 8, 543 + 884 + 1, 062

Exercise 1.5.13

(Solution on p. 81.)

10, 221 + 9, 016 + 11, 445

1.5.7 Exercises

For the following problems, perform the additions. If you can, check each sum with a calculator.

Exercise 1.5.14

(Solution on p. 81.)

14 + 5

Exercise 1.5.15

12 + 7

Exercise 1.5.16

(Solution on p. 81.)

46 + 2

Exercise 1.5.17

83 + 16

Exercise 1.5.18

(Solution on p. 81.)

77 + 21

Exercise 1.5.19

321

+ 42

Available for free at Connexions <http://cnx.org/content/col10615/1.4>

37

Exercise 1.5.20

(Solution on p. 82.)

916

+ 62

Exercise 1.5.21

104

+561

Exercise 1.5.22

(Solution on p. 82.)

265

+103

Exercise 1.5.23

552 + 237

Exercise 1.5.24

(Solution on p. 82.)

8, 521 + 4, 256

Exercise 1.5.25

16, 408

+ 3, 101

Exercise 1.5.26

(Solution on p. 82.)

16, 515

+42, 223

Exercise 1.5.27

616, 702 + 101, 161

Exercise 1.5.28

(Solution on p. 82.)

43, 156, 219 + 2, 013, 520

Exercise 1.5.29

17 + 6

Exercise 1.5.30

(Solution on p. 82.)

25 + 8

Exercise 1.5.31

84

+ 7

Exercise 1.5.32

(Solution on p. 82.)

75

+ 6

Exercise 1.5.33

36 + 48

Exercise 1.5.34

(Solution on p. 82.)

74 + 17

Exercise 1.5.35

486 + 58

Exercise 1.5.36

(Solution on p. 82.)

743 + 66

Exercise 1.5.37

381 + 88

Available for free at Connexions <http://cnx.org/content/col10615/1.4>

38

CHAPTER 1. ADDITION AND SUBTRACTION OF WHOLE NUMBERS

Exercise 1.5.38

(Solution on p. 82.)

687

+175

Exercise 1.5.39

931

+853

Exercise 1.5.40

(Solution on p. 82.)

1, 428 + 893

Exercise 1.5.41

12, 898 + 11, 925

Exercise 1.5.42

(Solution on p. 82.)

631, 464

+509, 740

Exercise 1.5.43

805, 996

+ 98, 516

Exercise 1.5.44

(Solution on p. 82.)

38, 428, 106

+522, 936, 005

Exercise 1.5.45

5, 288, 423, 100 + 16, 934, 785, 995

Exercise 1.5.46

(Solution on p. 82.)

98, 876, 678, 521, 402 + 843, 425, 685, 685, 658

Exercise 1.5.47

41 + 61 + 85 + 62

Exercise 1.5.48

(Solution on p. 82.)

21 + 85 + 104 + 9 + 15

Exercise 1.5.49

116

27

110

110

+

8

Exercise 1.5.50

(Solution on p. 82.)

75, 206

4, 152

+16, 007

Available for free at Connexions <http://cnx.org/content/col10615/1.4>

39

Exercise 1.5.51

8, 226

143

92, 015

8

487, 553

5, 218

Exercise 1.5.52

(Solution on p. 82.)

50, 006

1, 005

100, 300

20, 008

1, 000, 009

800, 800

Exercise 1.5.53

616

42, 018

1, 687

225

8, 623, 418

12, 506, 508

19

2, 121

195, 643

For the following problems, perform the additions and round to the nearest hundred.

Exercise 1.5.54

(Solution on p. 82.)

1, 468

2, 183

Exercise 1.5.55

928, 725

15, 685

Exercise 1.5.56

(Solution on p. 82.)

82, 006

3, 019, 528

Exercise 1.5.57

18, 621

5, 059

Available for free at Connexions <http://cnx.org/content/col10615/1.4>

40

CHAPTER 1. ADDITION AND SUBTRACTION OF WHOLE NUMBERS

Exercise 1.5.58

(Solution on p. 82.)

92

48

Exercise 1.5.59

16

37

Exercise 1.5.60

(Solution on p. 82.)

21

16

Exercise 1.5.61

11, 172

22, 749

12, 248

Exercise 1.5.62

(Solution on p. 82.)

240

280

210

310

Exercise 1.5.63

9, 573

101, 279

122, 581

For the next ve problems, replace the letter m with the whole number that will make the addition true.

Exercise 1.5.64

(Solution on p. 82.)

62

+

m

67

Exercise 1.5.65

106

+

m

113

Exercise 1.5.66

(Solution on p. 82.)

432

+

m

451

Exercise 1.5.67

803

+

m

830

Available for free at Connexions <http://cnx.org/content/col10615/1.4>

Image 42

41

Exercise 1.5.68

(Solution on p. 82.)

1, 893

+

m

1, 981

Exercise 1.5.69

The number of nursing and related care facilities in the United States in 1971 was 22,004. In 1978, the number was 18,722. What was the total number of facilities for both 1971 and 1978?

Exercise 1.5.70

(Solution on p. 83.)

The number of persons on food stamps in 1975, 1979, and 1980 was 19,179,000, 19,309,000, and 22,023,000, respectively. What was the total number of people on food stamps for the years 1975, 1979, and 1980?

Exercise 1.5.71

The enrollment in public and nonpublic schools in the years 1965, 1970, 1975, and 1984 was 54,394,000, 59,899,000, 61,063,000, and 55,122,000, respectively. What was the total enrollment for those years?

Exercise 1.5.72

(Solution on p. 83.)

The area of New England is 3,618,770 square miles. The area of the Mountain states is 863,563

square miles. The area of the South Atlantic is 278,926 square miles. The area of the Pacic states is 921,392 square miles. What is the total area of these regions?

Exercise 1.5.73

In 1960, the IRS received 1,188,000 corporate income tax returns. In 1965, 1,490,000 returns were received. In 1970, 1,747,000 returns were received. In 1972 1977, 1,890,000; 1,981,000; 2,043,000; 2,100,000; 2,159,000; and 2,329,000 returns were received, respectively. What was the total number of corporate tax returns received by the IRS during the years 1960, 1965, 1970, 1972 1977?

Exercise 1.5.74

(Solution on p. 83.)

Find the total number of scientists employed in 1974.

Exercise 1.5.75

Find the total number of sales for space vehicle systems for the years 1965-1980.

Available for free at Connexions <http://cnx.org/content/col10615/1.4>

Image 43

Image 44

Image 45

42

CHAPTER 1. ADDITION AND SUBTRACTION OF WHOLE NUMBERS

Exercise 1.5.76

(Solution on p. 83.)

Find the total baseball attendance for the years 1960-1980.

Exercise 1.5.77

Find the number of prosecutions of federal ocials for 1970-1980.

For the following problems, try to add the numbers mentally.

Available for free at Connexions <http://cnx.org/content/col10615/1.4>

43

Exercise 1.5.78

(Solution on p. 83.)

5

5

3

7

Exercise 1.5.79

8

2

6

4

Exercise 1.5.80

(Solution on p. 83.)

9

1

8

5

2

Exercise 1.5.81

5

2

5

8

3

7

Exercise 1.5.82

(Solution on p. 83.)

6

4

3

1

6

7

9

4

Exercise 1.5.83

20

30

Exercise 1.5.84

(Solution on p. 83.)

15

35

Available for free at Connexions <http://cnx.org/content/col10615/1.4>

44

CHAPTER 1. ADDITION AND SUBTRACTION OF WHOLE NUMBERS

Exercise 1.5.85

16

14

Exercise 1.5.86

(Solution on p. 83.)

23

27

Exercise 1.5.87

82

18

Exercise 1.5.88

(Solution on p. 83.)

36

14

1.5.7.1 Exercises for Review

Exercise 1.5.89

(Section 1.2) Each period of numbers has its own name. From right to left, what is the name of the fourth period?

Exercise 1.5.90

(Solution on p. 83.)

(Section 1.2) In the number 610,467, how many thousands are there?

Exercise 1.5.91

(Section 1.3) Write 8,840 as you would read it.

Exercise 1.5.92

(Solution on p. 83.)

(Section 1.4) Round 6,842 to the nearest hundred.

Exercise 1.5.93

(Section 1.4) Round 431,046 to the nearest million.

1.6 Subtraction of Whole Numbers6

1.6.1 Section Overview

• Subtraction

• Subtraction as the Opposite of Addition

• The Subtraction Process

• Subtraction Involving Borrowing

• Borrowing From Zero

• Calculators

6This content is available online at <http://cnx.org/content/m34784/1.5/>.

Available for free at Connexions <http://cnx.org/content/col10615/1.4>

Image 46

45

1.6.2 Subtraction

Subtraction

Subtraction is the process of determining the remainder when part of the total is removed.

Suppose the sum of two whole numbers is 11, and from 11 we remove 4. Using the number line to help our visualization, we see that if we are located at 11 and move 4 units to the left, and thus remove 4 units, we will be located at 7. Thus, 7 units remain when we remove 4 units from 11 units.

The Minus Symbol

The minus symbol (-) is used to indicate subtraction. For example, 11 − 4 indicates that 4 is to be subtracted from 11.

Minuend

The number immediately in front of or the minus symbol is called the minuend, and it represents the original number of units.

Subtrahend

The number immediately following or below the minus symbol is called the subtrahend, and it represents the number of units to be removed.

Dierence

The result of the subtraction is called the dierence of the two numbers. For example, in 11 − 4 = 7, 11 is the minuend, 4 is the subtrahend, and 7 is the dierence.

1.6.3 Subtraction as the Opposite of Addition

Subtraction can be thought of as the opposite of addition. We show this in the problems in Sample Set A.

1.6.3.1 Sample Set A

Example 1.31

8 − 5 = 3 since 3 + 5 = 8.

Example 1.32

9 − 3 = 6 since 6 + 3 = 9.

1.6.3.2 Practice Set A

Complete the following statements.

Exercise 1.6.1

(Solution on p. 83.)

7 − 5 =

since

+5 = 7.

Exercise 1.6.2

(Solution on p. 83.)

9 − 1 =

since

+1 = 9.

Exercise 1.6.3

(Solution on p. 83.)

17 − 8 =

since

+8 = 17.

Available for free at Connexions <http://cnx.org/content/col10615/1.4>

Image 47

46

CHAPTER 1. ADDITION AND SUBTRACTION OF WHOLE NUMBERS

1.6.4 The Subtraction Process

We’ll study the process of the subtraction of two whole numbers by considering the dierence between 48

and 35.

which we write as 13.

Example 1.33: The Process of Subtracting Whole Numbers

To subtract two whole numbers,

The process

1. Write the numbers vertically, placing corresponding positions in the same column.

48

−35

2. Subtract the digits in each column. Start at the right, in the ones position, and move to the left, placing the dierence at the bottom.

48

−35

13

1.6.4.1 Sample Set B

Perform the following subtractions.

Example 1.34

275

−142

133

5 – 2 = 3 .

7 – 4 = 3 .

2 – 1 = 1 .

Example 1.35

46, 042

− 1, 031

45, 011

2 – 1 = 1 .

4 – 3 = 1 .

0 – 0 = 0 .

6 – 1 = 5 .

4 – 0 = 4 .

Available for free at Connexions <http://cnx.org/content/col10615/1.4>

Image 48

47

Example 1.36

Find the dierence between 977 and 235.

Write the numbers vertically, placing the larger number on top. Line up the columns properly.

977

−235

742

The dierence between 977 and 235 is 742.

Example 1.37

In Keys County in 1987, there were 809 cable television installations. In Flags County in 1987, there were 1,159 cable television installations. How many more cable television installations were there in Flags County than in Keys County in 1987?

We need to determine the dierence between 1,159 and 809.

There were 350 more cable television installations in Flags County than in Keys County in 1987.

1.6.4.2 Practice Set B

Perform the following subtractions.

Exercise 1.6.4

(Solution on p. 83.)

534

−203

Exercise 1.6.5

(Solution on p. 83.)

857

− 43

Exercise 1.6.6

(Solution on p. 83.)

95, 628

−34, 510

Exercise 1.6.7

(Solution on p. 83.)

11, 005

− 1, 005

Exercise 1.6.8

(Solution on p. 83.)

Find the dierence between 88,526 and 26,412.

In each of these problems, each bottom digit is less than the corresponding top digit. This may not always be the case. We will examine the case where the bottom digit is greater than the corresponding top digit in the next section.

Available for free at Connexions <http://cnx.org/content/col10615/1.4>

Image 49

Image 50

Image 51

Image 52

Image 53

Image 54

48

CHAPTER 1. ADDITION AND SUBTRACTION OF WHOLE NUMBERS

1.6.5 Subtraction Involving Borrowing

Minuend and Subtrahend

It often happens in the subtraction of two whole numbers that a digit in the minuend (top number) will be less than the digit in the same position in the subtrahend (bottom number). This happens when we subtract 27 from 84.

84

−27

We do not have a name for 4 − 7. We need to rename 84 in order to continue. We’ll do so as follows: Our new name for 84 is 7 tens + 14 ones.

= 57

Notice that we converted 8 tens to 7 tens + 1 ten, and then we converted the 1 ten to 10 ones. We then had 14 ones and were able to perform the subtraction.

Borrowing

The process of borrowing (converting) is illustrated in the problems of Sample Set C.

1.6.5.1 Sample Set C

Example 1.38

1. Borrow 1 ten from the 8 tens. This leaves 7 tens.

2. Convert the 1 ten to 10 ones.

3. Add 10 ones to 4 ones to get 14 ones.

Example 1.39

1. Borrow 1 hundred from the 6 hundreds. This leaves 5 hundreds.

Available for free at Connexions <http://cnx.org/content/col10615/1.4>

Image 55

49

2. Convert the 1 hundred to 10 tens.

3. Add 10 tens to 7 tens to get 17 tens.

1.6.5.2 Practice Set C

Perform the following subtractions. Show the expanded form for the rst three problems.

Exercise 1.6.9

(Solution on p. 83.)

53

−35

Exercise 1.6.10

(Solution on p. 84.)

76

−28

Exercise 1.6.11

(Solution on p. 84.)

872

−565

Exercise 1.6.12

(Solution on p. 84.)

441

−356

Exercise 1.6.13

(Solution on p. 84.)

775

− 66

Exercise 1.6.14

(Solution on p. 84.)

5, 663

−2, 559

Borrowing More Than Once

Sometimes it is necessary to borrow more than once. This is shown in the problems in Section 1.6.5.3

(Sample Set D).

1.6.5.3 Sample Set D

Perform the Subtractions. Borrowing more than once if necessary

Example 1.40

1. Borrow 1 ten from the 4 tens. This leaves 3 tens.

2. Convert the 1 ten to 10 ones.

3. Add 10 ones to 1 one to get 11 ones. We can now perform 11 − 8.

4. Borrow 1 hundred from the 6 hundreds. This leaves 5 hundreds.

Available for free at Connexions <http://cnx.org/content/col10615/1.4>

Image 56

Image 57

50

CHAPTER 1. ADDITION AND SUBTRACTION OF WHOLE NUMBERS

5. Convert the 1 hundred to 10 tens.

6. Add 10 tens to 3 tens to get 13 tens.

7. Now 13 − 5 = 8.

8. 5 − 3 = 2.

Example 1.41

1. Borrow 1 ten from the 3 tens. This leaves 2 tens.

2. Convert the 1 ten to 10 ones.

3. Add 10 ones to 4 ones to get 14 ones. We can now perform 14 − 5.

4. Borrow 1 hundred from the 5 hundreds. This leaves 4 hundreds.

5. Convert the 1 hundred to 10 tens.

6. Add 10 tens to 2 tens to get 12 tens. We can now perform 12 − 8 = 4.

7. Finally, 4 − 0 = 4.

Example 1.42

71529

– 6952

After borrowing, we have

1.6.5.4 Practice Set D

Perform the following subtractions.

Exercise 1.6.15

(Solution on p. 84.)

526

−358

Exercise 1.6.16

(Solution on p. 84.)

63, 419

− 7, 779

Exercise 1.6.17

(Solution on p. 84.)

4, 312

−3, 123

Available for free at Connexions <http://cnx.org/content/col10615/1.4>

Image 58

Image 59

Image 60

51

1.6.6 Borrowing from Zero

It often happens in a subtraction problem that we have to borrow from one or more zeros. This occurs in problems such as

503

1. − 37

and

5000

2. − 37

We’ll examine each case.

Example 1.43: Borrowing from a single zero.

503

Consider the problem − 37

Since we do not have a name for 3 − 7, we must borrow from 0.

Since there are no tens to borrow, we must borrow 1 hundred. One hundred = 10 tens.

We can now borrow 1 ten from 10 tens (leaving 9 tens). One ten = 10 ones and 10 ones + 3 ones

= 13 ones.

Now we can suggest the following method for borrowing from a single zero.

Borrowing from a Single Zero

To borrow from a single zero,

1. Decrease the digit to the immediate left of zero by one.

2. Draw a line through the zero and make it a 10.

3. Proceed to subtract as usual.

1.6.6.1 Sample Set E

Example 1.44

Perform this subtraction.

503

− 37

The number 503 contains a single zero

Available for free at Connexions <http://cnx.org/content/col10615/1.4>

Image 61

Image 62

Image 63

Image 64

52

CHAPTER 1. ADDITION AND SUBTRACTION OF WHOLE NUMBERS

1. The number to the immediate left of 0 is 5. Decrease 5 by 1.

5 − 1 = 4

2. Draw a line through the zero and make it a 10.

3. Borrow from the 10 and proceed.

1 ten + 10 ones

10 ones + 3 ones = 13 ones

1.6.6.2 Practice Set E

Perform each subtraction.

Exercise 1.6.18

(Solution on p. 84.)

906

− 18

Exercise 1.6.19

(Solution on p. 84.)

5102

− 559

Exercise 1.6.20

(Solution on p. 85.)

9055

− 386

Example 1.45: Borrowing from a group of zeros

5000

Consider the problem − 37

In this case, we have a group of zeros.

Since we cannot borrow any tens or hundreds, we must borrow 1 thousand. One thousand = 10

hundreds.

We can now borrow 1 hundred from 10 hundreds. One hundred = 10 tens.

Available for free at Connexions <http://cnx.org/content/col10615/1.4>

Image 65

Image 66

Image 67

Image 68

53

We can now borrow 1 ten from 10 tens. One ten = 10 ones.

From observations made in this procedure we can suggest the following method for borrowing from a group of zeros.

Borrowing from a Group of zeros

To borrow from a group of zeros,

1. Decrease the digit to the immediate left of the group of zeros by one.

2. Draw a line through each zero in the group and make it a 9, except the rightmost zero, make it 10.

3. Proceed to subtract as usual.

1.6.6.3 Sample Set F

Perform each subtraction.

Example 1.46

40, 000

125

The number 40,000 contains a group of zeros.

1. The number to the immediate left of the group is 4. Decrease 4 by 1.

4 − 1 = 3

2. Make each 0, except the rightmost one, 9. Make the rightmost 0 a 10.

3. Subtract as usual.

Example 1.47

8, 000, 006

41, 107

The number 8,000,006 contains a group of zeros.

1. The number to the immediate left of the group is 8. Decrease 8 by 1. 8 − 1 = 7

2. Make each zero, except the rightmost one, 9. Make the rightmost 0 a 10.

Available for free at Connexions <http://cnx.org/content/col10615/1.4>

Image 69

Image 70

54

CHAPTER 1. ADDITION AND SUBTRACTION OF WHOLE NUMBERS

3. To perform the subtraction, we’ll need to borrow from the ten.

1 ten = 10 ones

10 ones + 6 ones = 16 ones

1.6.6.4 Practice Set F

Perform each subtraction.

Exercise 1.6.21

(Solution on p. 85.)

21, 007

− 4, 873

Exercise 1.6.22

(Solution on p. 85.)

10, 004

− 5, 165

Exercise 1.6.23

(Solution on p. 85.)

16, 000, 000

201, 060

1.6.7 Calculators

In practice, calculators are used to nd the dierence between two whole numbers.

1.6.7.1 Sample Set G

Find the dierence between 1006 and 284.

Display Reads

Type 1006 1006

Press −

1006

Type 284

284

Press =

722

Table 1.26

Available for free at Connexions <http://cnx.org/content/col10615/1.4>

55

The dierence between 1006 and 284 is 722.

(What happens if you type 284 rst and then 1006? We’ll study such numbers in Chapter 10.) 1.6.7.2 Practice Set G

Exercise 1.6.24

(Solution on p. 85.)

Use a calculator to nd the dierence between 7338 and 2809.

Exercise 1.6.25

(Solution on p. 85.)

Use a calculator to nd the dierence between 31,060,001 and 8,591,774.

1.6.8 Exercises

For the following problems, perform the subtractions. You may check each dierence with a calculator.

Exercise 1.6.26

(Solution on p. 85.)

15

− 8

Exercise 1.6.27

19

− 8

Exercise 1.6.28

(Solution on p. 85.)

11

− 5

Exercise 1.6.29

14

− 6

Exercise 1.6.30

(Solution on p. 85.)

12

− 9

Exercise 1.6.31

56

−12

Exercise 1.6.32

(Solution on p. 85.)

74

−33

Exercise 1.6.33

80

−61

Exercise 1.6.34

(Solution on p. 85.)

350

−141

Available for free at Connexions <http://cnx.org/content/col10615/1.4>

56

CHAPTER 1. ADDITION AND SUBTRACTION OF WHOLE NUMBERS

Exercise 1.6.35

800

−650

Exercise 1.6.36

(Solution on p. 85.)

35, 002

−14, 001

Exercise 1.6.37

5, 000, 566

−2, 441, 326

Exercise 1.6.38

(Solution on p. 85.)

400, 605

−121, 352

Exercise 1.6.39

46, 400

− 2, 012

Exercise 1.6.40

(Solution on p. 85.)

77, 893

421

Exercise 1.6.41

42

−18

Exercise 1.6.42

(Solution on p. 85.)

51

−27

Exercise 1.6.43

622

− 88

Exercise 1.6.44

(Solution on p. 85.)

261

− 73

Exercise 1.6.45

242

−158

Exercise 1.6.46

(Solution on p. 85.)

3, 422

−1, 045

Exercise 1.6.47

5, 565

−3, 985

Available for free at Connexions <http://cnx.org/content/col10615/1.4>

57

Exercise 1.6.48

(Solution on p. 85.)

42, 041

−15, 355

Exercise 1.6.49

304, 056

− 20, 008

Exercise 1.6.50

(Solution on p. 85.)

64, 000, 002

856, 743

Exercise 1.6.51

4, 109

− 856

Exercise 1.6.52

(Solution on p. 85.)

10, 113

− 2, 079

Exercise 1.6.53

605

− 77

Exercise 1.6.54

(Solution on p. 85.)

59

−26

Exercise 1.6.55

36, 107

− 8, 314

Exercise 1.6.56

(Solution on p. 85.)

92, 526, 441, 820

−59, 914, 805, 253

Exercise 1.6.57

1, 605

− 881

Exercise 1.6.58

(Solution on p. 85.)

30, 000

−26, 062

Exercise 1.6.59

600

−216

Exercise 1.6.60

(Solution on p. 85.)

9, 000, 003

726, 048

For the following problems, perform each subtraction.

Available for free at Connexions <http://cnx.org/content/col10615/1.4>

58

CHAPTER 1. ADDITION AND SUBTRACTION OF WHOLE NUMBERS

Exercise 1.6.61

Subtract 63 from 92.

Hint:

The word “from” means “beginning at.” Thus, 63 from 92 means beginning at 92, or 92 − 63.

Exercise 1.6.62

(Solution on p. 85.)

Subtract 35 from 86.

Exercise 1.6.63

Subtract 382 from 541.

Exercise 1.6.64

(Solution on p. 86.)

Subtract 1,841 from 5,246.

Exercise 1.6.65

Subtract 26,082 from 35,040.

Exercise 1.6.66

(Solution on p. 86.)

Find the dierence between 47 and 21.

Exercise 1.6.67

Find the dierence between 1,005 and 314.

Exercise 1.6.68

(Solution on p. 86.)

Find the dierence between 72,085 and 16.

Exercise 1.6.69

Find the dierence between 7,214 and 2,049.

Exercise 1.6.70

(Solution on p. 86.)

Find the dierence between 56,108 and 52,911.

Exercise 1.6.71

How much bigger is 92 than 47?

Exercise 1.6.72

(Solution on p. 86.)

How much bigger is 114 than 85?

Exercise 1.6.73

How much bigger is 3,006 than 1,918?

Exercise 1.6.74

(Solution on p. 86.)

How much bigger is 11,201 than 816?

Exercise 1.6.75

How much bigger is 3,080,020 than 1,814,161?

Exercise 1.6.76

(Solution on p. 86.)

In Wichita, Kansas, the sun shines about 74% of the time in July and about 59% of the time in November. How much more of the time (in percent) does the sun shine in July than in November?

Exercise 1.6.77

The lowest temperature on record in Concord, New Hampshire in May is 21 ◦F, and in July it is 35 ◦F. What is the dierence in these lowest temperatures?

Exercise 1.6.78

(Solution on p. 86.)

In 1980, there were 83,000 people arrested for prostitution and commercialized vice and 11,330,000

people arrested for driving while intoxicated. How many more people were arrested for drunk driving than for prostitution?

Exercise 1.6.79

In 1980, a person with a bachelor’s degree in accounting received a monthly salary oer of $1,293, and a person with a marketing degree a monthly salary oer of $1,145. How much more was oered to the person with an accounting degree than the person with a marketing degree?

Available for free at Connexions <http://cnx.org/content/col10615/1.4>

Image 71

59

Exercise 1.6.80

(Solution on p. 86.)

In 1970, there were about 793 people per square mile living in Puerto Rico, and 357 people per square mile living in Guam. How many more people per square mile were there in Puerto Rico than Guam?

Exercise 1.6.81

The 1980 population of Singapore was 2,414,000 and the 1980 population of Sri Lanka was 14,850,000. How many more people lived in Sri Lanka than in Singapore in 1980?

Exercise 1.6.82

(Solution on p. 86.)

In 1977, there were 7,234,000 hospitals in the United States and 64,421,000 in Mainland China.

How many more hospitals were there in Mainland China than in the United States in 1977?

Exercise 1.6.83

In 1978, there were 3,095,000 telephones in use in Poland and 4,292,000 in Switzerland. How many more telephones were in use in Switzerland than in Poland in 1978?

For the following problems, use the corresponding graphs to solve the problems.

Exercise 1.6.84

(Solution on p. 86.)

How many more life scientists were there in 1974 than mathematicians? (this image) Exercise 1.6.85

How many more social, psychological, mathematical, and environmental scientists were there than life, physical, and computer scientists? (this image)

Exercise 1.6.86

(Solution on p. 86.)

How many more prosecutions were there in 1978 than in 1974? (this image) Exercise 1.6.87

How many more prosecutions were there in 1976-1980 than in 1970-1975? (this image) Available for free at Connexions <http://cnx.org/content/col10615/1.4>

Image 72

Image 73

60

CHAPTER 1. ADDITION AND SUBTRACTION OF WHOLE NUMBERS

Exercise 1.6.88

(Solution on p. 86.)

How many more dry holes were drilled in 1960 than in 1975? (this image) Exercise 1.6.89

How many more dry holes were drilled in 1960, 1965, and 1970 than in 1975, 1978 and 1979? (this image)

For the following problems, replace the [U+2610] with the whole number that will make the subtraction true.

Exercise 1.6.90

(Solution on p. 86.)

14

3

Available for free at Connexions <http://cnx.org/content/col10615/1.4>

61

Exercise 1.6.91

21

14

Exercise 1.6.92

(Solution on p. 86.)

35

25

Exercise 1.6.93

16

9

Exercise 1.6.94

(Solution on p. 86.)

28

16

For the following problems, nd the solutions.

Exercise 1.6.95

Subtract 42 from the sum of 16 and 56.

Exercise 1.6.96

(Solution on p. 86.)

Subtract 105 from the sum of 92 and 89.

Exercise 1.6.97

Subtract 1,127 from the sum of 2,161 and 387.

Exercise 1.6.98

(Solution on p. 86.)

Subtract 37 from the dierence between 263 and 175.

Exercise 1.6.99

Subtract 1,109 from the dierence between 3,046 and 920.

Exercise 1.6.100

(Solution on p. 86.)

Add the dierence between 63 and 47 to the difference between 55 and 11.

Exercise 1.6.101

Add the dierence between 815 and 298 to the dierence between 2,204 and 1,016.

Exercise 1.6.102

(Solution on p. 86.)

Subtract the dierence between 78 and 43 from the sum of 111 and 89.

Exercise 1.6.103

Subtract the dierence between 18 and 7 from the sum of the dierences between 42 and 13, and 81 and 16.

Exercise 1.6.104

(Solution on p. 86.)

Find the dierence between the dierences of 343 and 96, and 521 and 488.

Available for free at Connexions <http://cnx.org/content/col10615/1.4>

Image 74

62

CHAPTER 1. ADDITION AND SUBTRACTION OF WHOLE NUMBERS

1.6.8.1 Exercises for Review

Exercise 1.6.105

(Section 1.2) In the number 21,206, how many hundreds are there?

Exercise 1.6.106

(Solution on p. 86.)

(Section 1.2) Write a three-digit number that has a zero in the ones position.

Exercise 1.6.107

(Section 1.2) How many three-digit whole numbers are there?

Exercise 1.6.108

(Solution on p. 86.)

(Section 1.4) Round 26,524,016 to the nearest million.

Exercise 1.6.109

(Section 1.5) Find the sum of 846 + 221 + 116.

1.7 Properties of Addition7

1.7.1 Section Overview

• The Commutative Property of Addition

• The Associative Property of Addition

• The Additive Identity

We now consider three simple but very important properties of addition.

1.7.2 The Commutative Property of Addition

Commutative Property of Addition

If two whole numbers are added in any order, the sum will not change.

1.7.2.1 Sample Set A

Example 1.48

Add the whole numbers

8 + 5 = 13

5 + 8 = 13

The numbers 8 and 5 can be added in any order. Regardless of the order they are added, the sum is 13.

7This content is available online at <http://cnx.org/content/m34802/1.5/>.

Available for free at Connexions <http://cnx.org/content/col10615/1.4>

Image 75

Image 76

Image 77

Image 78

Image 79

63

1.7.2.2 Practice Set A

Exercise 1.7.1

(Solution on p. 86.)

Use the commutative property of addition to nd the sum of 12 and 41 in two dierent ways.

Exercise 1.7.2

(Solution on p. 86.)

Add the whole numbers

1.7.3 The Associative Property of Addition

Associative Property of Addition

If three whole numbers are to be added, the sum will be the same if the rst two are added rst, then that sum is added to the third, or, the second two are added rst, and that sum is added to the rst.

Using Parentheses

It is a common mathematical practice to use parentheses to show which pair of numbers we wish to combine rst.

1.7.3.1 Sample Set B

Example 1.49

Add the whole numbers.

1.7.3.2 Practice Set B

Exercise 1.7.3

(Solution on p. 87.)

Use the associative property of addition to add the following whole numbers two dierent ways.

Exercise 1.7.4

(Solution on p. 87.)

Available for free at Connexions <http://cnx.org/content/col10615/1.4>

Image 80

Image 81

Image 82

Image 83

Image 84

64

CHAPTER 1. ADDITION AND SUBTRACTION OF WHOLE NUMBERS

1.7.4 The Additive Identity

0 Is the Additive Identity

The whole number 0 is called the additive identity, since when it is added to any whole number, the sum is identical to that whole number.

1.7.4.1 Sample Set C

Example 1.50

Add the whole numbers.

29 + 0 = 29

0 + 29 = 29

Zero added to 29 does not change the identity of 29.

1.7.4.2 Practice Set C

Add the following whole numbers.

Exercise 1.7.5

(Solution on p. 87.)

Exercise 1.7.6

(Solution on p. 87.)

Suppose we let the letter x represent a choice for some whole number. For the rst two problems, nd the sums. For the third problem, nd the sum provided we now know that x represents the whole number 17.

Exercise 1.7.7

(Solution on p. 87.)

Exercise 1.7.8

(Solution on p. 87.)

Available for free at Connexions <http://cnx.org/content/col10615/1.4>

Image 85

Image 86

Image 87

Image 88

Image 89

Image 90

Image 91

Image 92

65

Exercise 1.7.9

(Solution on p. 87.)

1.7.5 Exercises

For the following problems, add the numbers in two ways.

Exercise 1.7.10

(Solution on p. 87.)

Exercise 1.7.11

Exercise 1.7.12

(Solution on p. 87.)

Exercise 1.7.13

Exercise 1.7.14

(Solution on p. 87.)

Exercise 1.7.15

Exercise 1.7.16

(Solution on p. 87.)

Available for free at Connexions <http://cnx.org/content/col10615/1.4>

Image 93

Image 94

Image 95

Image 96

Image 97

Image 98

Image 99

Image 100

66

CHAPTER 1. ADDITION AND SUBTRACTION OF WHOLE NUMBERS

Exercise 1.7.17

Exercise 1.7.18

(Solution on p. 87.)

Exercise 1.7.19

Exercise 1.7.20

(Solution on p. 87.)

Exercise 1.7.21

Exercise 1.7.22

(Solution on p. 87.)

Exercise 1.7.23

Exercise 1.7.24

(Solution on p. 87.)

For the following problems, show that the pairs of quantities yield the same sum.

Exercise 1.7.25

(11 + 27) + 9 and 11 + (27 + 9)

Exercise 1.7.26

(Solution on p. 87.)

(80 + 52) + 6 and 80 + (52 + 6)

Available for free at Connexions <http://cnx.org/content/col10615/1.4>

67

Exercise 1.7.27

(114 + 226) + 108 and 114 + (226 + 108)

Exercise 1.7.28

(Solution on p. 87.)

(731 + 256) + 171 and 731 + (256 + 171)

Exercise 1.7.29

The fact that (a rst number + a second number) + third number = a rst number + (a second number + a third number) is an example of the

property of addition.

Exercise 1.7.30

(Solution on p. 87.)

The fact that 0 + any number = that particular number is an example of the property of addition.

Exercise 1.7.31

The fact that a rst number + a second number = a second number + a rst number is an example of the

property of addition.

Exercise 1.7.32

(Solution on p. 87.)

Use the numbers 15 and 8 to illustrate the commutative property of addition.

Exercise 1.7.33

Use the numbers 6, 5, and 11 to illustrate the associative property of addition.

Exercise 1.7.34

(Solution on p. 87.)

The number zero is called the additive identity. Why is the term identity so appropriate?

1.7.5.1 Exercises for Review

Exercise 1.7.35

(Section 1.2) How many hundreds in 46,581?

Exercise 1.7.36

(Solution on p. 87.)

(Section 1.3) Write 2,218 as you would read it.

Exercise 1.7.37

(Section 1.4) Round 506,207 to the nearest thousand.

Exercise 1.7.38

(Solution on p. 87.)

482

(Section 1.5) Find the sum of + 68

Exercise 1.7.39

3, 318

(Section 1.6) Find the dierence: − 429

Available for free at Connexions <http://cnx.org/content/col10615/1.4>

Image 101

Image 102

68

CHAPTER 1. ADDITION AND SUBTRACTION OF WHOLE NUMBERS

1.8 Summary of Key Concepts8

1.8.1 Summary of Key Concepts

Number / Numeral (Section 1.2)

A number is a concept. It exists only in the mind. A numeral is a symbol that represents a number. It is customary not to distinguish between the two (but we should remain aware of the dierence).

Hindu-Arabic Numeration System (Section 1.2)

In our society, we use the Hindu-Arabic numeration system. It was invented by the Hindus shortly before the third century and popularized by the Arabs about a thousand years later.

Digits (Section 1.2)

The numbers 0, 1, 2, 3, 4, 5, 6, 7, 8, 9 are called digits.

Base Ten Positional System (Section 1.2)

The Hindu-Arabic numeration system is a positional number system with base ten. Each position has value that is ten times the value of the position to its right.

Commas / Periods (Section 1.2)

Commas are used to separate digits into groups of three. Each group of three is called a period. Each period has a name. From right to left, they are ones, thousands, millions, billions, etc.

Whole Numbers (Section 1.2)

A whole number is any number that is formed using only the digits (0, 1, 2, 3, 4, 5, 6, 7, 8, 9).

Number Line (Section 1.2)

The number line allows us to visually display the whole numbers.

Graphing (Section 1.2)

Graphing a whole number is a term used for visually displaying the whole number. The graph of 4 appears below.

Reading Whole Numbers (Section 1.3)

To express a whole number as a verbal phrase:

1. Begin at the right and, working right to left, separate the number into distinct periods by inserting commas every three digits.

2. Begin at the left, and read each period individually.

Writing Whole Numbers (Section 1.3)

To rename a number that is expressed in words to a number expressed in digits: 8This content is available online at <http://cnx.org/content/m34798/1.3/>.

Available for free at Connexions <http://cnx.org/content/col10615/1.4>

Image 103

69

1. Notice that a number expressed as a verbal phrase will have its periods set o by commas.

2. Start at the beginning of the sentence, and write each period of numbers individually.

3. Use commas to separate periods, and combine the periods to form one number.

Rounding (Section 1.4)

Rounding is the process of approximating the number of a group of objects by mentally “seeing” the collection as occurring in groups of tens, hundreds, thousands, etc.

Addition (Section 1.5)

Addition is the process of combining two or more objects (real or intuitive) to form a new, third object, the total, or sum.

Addends / Sum (Section 1.5)

In addition, the numbers being added are called addends and the result, or total, the sum.

Subtraction (Section 1.6)

Subtraction is the process of determining the remainder when part of the total is removed.

Minuend / Subtrahend Dierence (Section 1.6)

Commutative Property of Addition (Section 1.7)

If two whole numbers are added in either of two orders, the sum will not change.

3 + 5 = 5 + 3

Associative Property of Addition (Section 1.7)

If three whole numbers are to be added, the sum will be the same if the rst two are added and that sum is then added to the third, or if the second two are added and the rst is added to that sum.

(3 + 5) + 2 = 3 + (5 + 2)

Parentheses in Addition (Section 1.7)

Parentheses in addition indicate which numbers are to be added rst.

Additive Identity (Section 1.7)

The whole number 0 is called the additive identity since, when it is added to any particular whole number, the sum is identical to that whole number.

0 + 7 = 7

7 + 0 = 7

1.9 Exercise Supplement9

1.9.1 Exercise Supplement

For problems 1-35, nd the sums and dierences.

Exercise 1.9.1

(Solution on p. 87.)

908

+ 29

9This content is available online at <http://cnx.org/content/m34800/1.5/>.

Available for free at Connexions <http://cnx.org/content/col10615/1.4>

70

CHAPTER 1. ADDITION AND SUBTRACTION OF WHOLE NUMBERS

Exercise 1.9.2

529

+161

Exercise 1.9.3

(Solution on p. 87.)

549

+ 16

Exercise 1.9.4

726

+892

Exercise 1.9.5

(Solution on p. 87.)

390

+169

Exercise 1.9.6

166

+660

Exercise 1.9.7

(Solution on p. 88.)

391

+951

Exercise 1.9.8

48

+36

Exercise 1.9.9

(Solution on p. 88.)

1, 103

+ 898

Exercise 1.9.10

1, 642

+ 899

Exercise 1.9.11

(Solution on p. 88.)

807

+1, 156

Exercise 1.9.12

80, 349

+ 2, 679

Exercise 1.9.13

(Solution on p. 88.)

70, 070

+ 9, 386

Exercise 1.9.14

90, 874

+ 2, 945

Available for free at Connexions <http://cnx.org/content/col10615/1.4>

71

Exercise 1.9.15

(Solution on p. 88.)

45, 292

+51, 661

Exercise 1.9.16

1, 617

+54, 923

Exercise 1.9.17

(Solution on p. 88.)

702, 607

+ 89, 217

Exercise 1.9.18

6, 670, 006

+

2, 495

Exercise 1.9.19

(Solution on p. 88.)

267

+8, 034

Exercise 1.9.20

7, 007

+11, 938

Exercise 1.9.21

(Solution on p. 88.)

131, 294

+

9, 087

Exercise 1.9.22

5, 292

+

161

Exercise 1.9.23

(Solution on p. 88.)

17, 260

+58, 964

Exercise 1.9.24

7, 006

−5, 382

Exercise 1.9.25

(Solution on p. 88.)

7, 973

−3, 018

Exercise 1.9.26

16, 608

− 1, 660

Exercise 1.9.27

(Solution on p. 88.)

209, 527

23, 916

Available for free at Connexions <http://cnx.org/content/col10615/1.4>

72

CHAPTER 1. ADDITION AND SUBTRACTION OF WHOLE NUMBERS

Exercise 1.9.28

584

−226

Exercise 1.9.29

(Solution on p. 88.)

3, 313

−1, 075

Exercise 1.9.30

458

−122

Exercise 1.9.31

(Solution on p. 88.)

1, 007

+ 331

Exercise 1.9.32

16, 082

+ 2, 013

Exercise 1.9.33

(Solution on p. 88.)

926

− 48

Exercise 1.9.34

736

+5, 869

Exercise 1.9.35

(Solution on p. 88.)

676, 504

− 58, 277

For problems 36-39, add the numbers.

Exercise 1.9.36

769

795

298

746

Exercise 1.9.37

(Solution on p. 88.)

554

184

883

Available for free at Connexions <http://cnx.org/content/col10615/1.4>

73

Exercise 1.9.38

30, 188

79, 731

16, 600

66, 085

39, 169

95, 170

Exercise 1.9.39

(Solution on p. 88.)

2, 129

6, 190

17, 044

30, 447

292

41

428, 458

For problems 40-50, combine the numbers as indicated.

Exercise 1.9.40

2, 957 + 9, 006

Exercise 1.9.41

(Solution on p. 88.)

19, 040 + 813

Exercise 1.9.42

350, 212 + 14, 533

Exercise 1.9.43

(Solution on p. 88.)

970 + 702 + 22 + 8

Exercise 1.9.44

3, 704 + 2, 344 + 429 + 10, 374 + 74

Exercise 1.9.45

(Solution on p. 88.)

874 + 845 + 295 − 900

Exercise 1.9.46

904 + 910 − 881

Exercise 1.9.47

(Solution on p. 88.)

521 + 453 − 334 + 600

Exercise 1.9.48

892 − 820 − 9

Exercise 1.9.49

(Solution on p. 88.)

159 + 4, 085 − 918 − 608

Exercise 1.9.50

2, 562 + 8, 754 − 393 − 385 − 910

For problems 51-63, add and subtract as indicated.

Exercise 1.9.51

(Solution on p. 88.)

Subtract 671 from 8,027.

Available for free at Connexions <http://cnx.org/content/col10615/1.4>

Image 104

74

CHAPTER 1. ADDITION AND SUBTRACTION OF WHOLE NUMBERS

Exercise 1.9.52

Subtract 387 from 6,342.

Exercise 1.9.53

(Solution on p. 88.)

Subtract 2,926 from 6,341.

Exercise 1.9.54

Subtract 4,355 from the sum of 74 and 7,319.

Exercise 1.9.55

(Solution on p. 88.)

Subtract 325 from the sum of 7,188 and 4,964.

Exercise 1.9.56

Subtract 496 from the dierence of 60,321 and 99.

Exercise 1.9.57

(Solution on p. 89.)

Subtract 20,663 from the dierence of 523,150 and 95,225.

Exercise 1.9.58

Add the dierence of 843 and 139 to the dierence of 4,450 and 839.

Exercise 1.9.59

(Solution on p. 89.)

Add the dierence of 997,468 and 292,513 to the dierence of 22,140 and 8,617.

Exercise 1.9.60

Subtract the dierence of 8,412 and 576 from the sum of 22,140 and 8,617.

Exercise 1.9.61

(Solution on p. 89.)

Add the sum of 2,273, 3,304, 847, and 16 to the dierence of 4,365 and 864.

Exercise 1.9.62

Add the sum of 19,161, 201, 166,127, and 44 to the dierence of the sums of 161, 2,455, and 85, and 21, 26, 48, and 187.

Exercise 1.9.63

(Solution on p. 89.)

Is the sum of 626 and 1,242 the same as the sum of 1,242 and 626? Justify your claim.

1.10 Prociency Exam10

1.10.1 Prociency Exam

Exercise 1.10.1

(Solution on p. 89.)

(Section 1.2) What is the largest digit?

Exercise 1.10.2

(Solution on p. 89.)

(Section 1.2) In the Hindu-Arabic number system, each period has three values assigned to it.

These values are the same for each period. From right to left, what are they?

Exercise 1.10.3

(Solution on p. 89.)

(Section 1.2) In the number 42,826, how many hundreds are there?

Exercise 1.10.4

(Solution on p. 89.)

(Section 1.2) Is there a largest whole number? If so, what is it?

Exercise 1.10.5

(Solution on p. 89.)

(Section 1.2) Graph the following whole numbers on the number line: 2, 3, 5.

10This content is available online at <http://cnx.org/content/m34805/1.4/>.

Available for free at Connexions <http://cnx.org/content/col10615/1.4>

75

Exercise 1.10.6

(Solution on p. 89.)

(Section 1.3) Write the number 63,425 as you would read it aloud.

Exercise 1.10.7

(Solution on p. 89.)

(Section 1.3) Write the number eighteen million, three hundred fty-nine thousand, seventy-two.

Exercise 1.10.8

(Solution on p. 89.)

(Section 1.4) Round 427 to the nearest hundred.

Exercise 1.10.9

(Solution on p. 89.)

(Section 1.4) Round 18,995 to the nearest ten.

Exercise 1.10.10

(Solution on p. 89.)

(Section 1.4) Round to the most reasonable digit: During a semester, a mathematics instructor uses 487 pieces of chalk.

For problems 11-17, nd the sums and dierences.

Exercise 1.10.11

(Solution on p. 89.)

627

(Section 1.5) + 48

Exercise 1.10.12

(Solution on p. 89.)

(Section 1.5) 3106 + 921

Exercise 1.10.13

(Solution on p. 89.)

152

(Section 1.5) + 36

Exercise 1.10.14

(Solution on p. 89.)

5, 189

6, 189

(Section 1.5)

4, 122

+8, 001

Exercise 1.10.15

(Solution on p. 89.)

(Section 1.5) 21 + 16 + 42 + 11

Exercise 1.10.16

(Solution on p. 89.)

(Section 1.6) 520 − 216

Exercise 1.10.17

(Solution on p. 89.)

80, 001

(Section 1.6) − 9,878

Exercise 1.10.18

(Solution on p. 89.)

(Section 1.6) Subtract 425 from 816.

Exercise 1.10.19

(Solution on p. 89.)

(Section 1.6) Subtract 712 from the sum of 507 and 387.

Exercise 1.10.20

(Solution on p. 89.)

(Section 1.7) Is the sum of 219 and 412 the same as the sum of 412 and 219? If so, what makes it so?

Available for free at Connexions <http://cnx.org/content/col10615/1.4>

Image 105

Image 106

76

CHAPTER 1. ADDITION AND SUBTRACTION OF WHOLE NUMBERS

Solutions to Exercises in Chapter 1

Solution to Exercise 1.2.1 (p. 10)

Yes. Letters are symbols. Taken as a collection (a written word), they represent a number.

Solution to Exercise 1.2.2 (p. 12)

ve thousand

Solution to Exercise 1.2.3 (p. 12)

four hundred billion

Solution to Exercise 1.2.4 (p. 12)

zero tens, or zero

Solution to Exercise 1.2.5 (p. 13)

Solution to Exercise 1.2.6 (p. 13)

4, 5, 6, 113, 978

Solution to Exercise 1.2.7 (p. 14)

concept

Solution to Exercise 1.2.9 (p. 14)

Yes, since it is a symbol that represents a number.

Solution to Exercise 1.2.11 (p. 14)

positional; 10

Solution to Exercise 1.2.13 (p. 14)

ones, tens, hundreds

Solution to Exercise 1.2.15 (p. 14)

4

Solution to Exercise 1.2.17 (p. 14)

0

Solution to Exercise 1.2.19 (p. 14)

0

Solution to Exercise 1.2.21 (p. 14)

ten thousand

Solution to Exercise 1.2.23 (p. 14)

6 ten millions = 60 million

Solution to Exercise 1.2.25 (p. 14)

1,340 (answers may vary)

Solution to Exercise 1.2.27 (p. 15)

900

Solution to Exercise 1.2.29 (p. 15)

yes; zero

Solution to Exercise 1.2.31 (p. 15)

graphing

Solution to Exercise 1.2.33 (p. 15)

Solution to Exercise 1.2.35 (p. 15)

61, 99, 100, 102

Solution to Exercise 1.3.1 (p. 17)

Twelve thousand, ve hundred forty-two

Solution to Exercise 1.3.2 (p. 17)

One hundred one million, seventy-four thousand, three

Available for free at Connexions <http://cnx.org/content/col10615/1.4>

77

Solution to Exercise 1.3.3 (p. 17)

One million, eight

Solution to Exercise 1.3.4 (p. 18)

103,025

Solution to Exercise 1.3.5 (p. 18)

6,040,007

Solution to Exercise 1.3.6 (p. 18)

20,003,080,109,402

Solution to Exercise 1.3.7 (p. 18)

80,000,000,035

Solution to Exercise 1.3.8 (p. 18)

nine hundred twelve

Solution to Exercise 1.3.10 (p. 19)

one thousand, four hundred ninety-one

Solution to Exercise 1.3.12 (p. 19)

thirty-ve thousand, two hundred twenty-three

Solution to Exercise 1.3.14 (p. 19)

four hundred thirty-seven thousand, one hundred ve

Solution to Exercise 1.3.16 (p. 19)

eight million, one thousand, one

Solution to Exercise 1.3.18 (p. 19)

seven hundred seventy million, three hundred eleven thousand, one hundred one Solution to Exercise 1.3.20 (p. 19)

one hundred six billion, one hundred million, one thousand ten

Solution to Exercise 1.3.22 (p. 19)

eight hundred billion, eight hundred thousand

Solution to Exercise 1.3.24 (p. 19)

four; one thousand, four hundred sixty

Solution to Exercise 1.3.26 (p. 19)

twenty billion

Solution to Exercise 1.3.28 (p. 19)

four hundred twelve; fty-two; twenty-one thousand, four hundred twenty-four Solution to Exercise 1.3.30 (p. 19)

one thousand, nine hundred seventy-nine; eighty-ve thousand; two million, nine hundred ve thousand Solution to Exercise 1.3.32 (p. 19)

one thousand, nine hundred eighty; two hundred seventeen

Solution to Exercise 1.3.34 (p. 20)

one thousand, nine hundred eighty one; one million, nine hundred fty-six thousand Solution to Exercise 1.3.36 (p. 20)

one thousand, nine hundred eighty; thirteen thousand, one hundred

Solution to Exercise 1.3.38 (p. 20)

twelve million, six hundred thirty thousand

Solution to Exercise 1.3.40 (p. 20)

681

Solution to Exercise 1.3.42 (p. 20)

7,201

Solution to Exercise 1.3.44 (p. 20)

512,003

Solution to Exercise 1.3.46 (p. 20)

35,007,101

Available for free at Connexions <http://cnx.org/content/col10615/1.4>

78

CHAPTER 1. ADDITION AND SUBTRACTION OF WHOLE NUMBERS

Solution to Exercise 1.3.48 (p. 20)

16,000,059,004

Solution to Exercise 1.3.50 (p. 20)

23,000,000,000

Solution to Exercise 1.3.52 (p. 20)

100,000,000,000,001

Solution to Exercise 1.3.54 (p. 21)

4

Solution to Exercise 1.3.56 (p. 21)

yes, zero

Solution to Exercise 1.4.1 (p. 24)

3400

Solution to Exercise 1.4.2 (p. 24)

27,000

Solution to Exercise 1.4.3 (p. 24)

31,000,000

Solution to Exercise 1.4.4 (p. 24)

0

Solution to Exercise 1.4.5 (p. 24)

60,000

Solution to Exercise 1.4.6 (p. 24)

hundred thousand ten thousand million

1,600

2000

0

0

Table 1.27

Solution to Exercise 1.4.8 (p. 25)

Hundred thousand ten thousand million

91,800

92,000

90,000

0

Table 1.28

Solution to Exercise 1.4.10 (p. 25)

hundred thousand ten thousand million

200

0

0

0

Table 1.29

Solution to Exercise 1.4.12 (p. 25)

hundred thousand ten thousand million

900

1,000

0

0

Available for free at Connexions <http://cnx.org/content/col10615/1.4>

79

Table 1.30

Solution to Exercise 1.4.14 (p. 26)

hundred thousand ten thousand million

900

1,000

0

0

Table 1.31

Solution to Exercise 1.4.16 (p. 26)

hundred thousand ten thousand million

1,000

1,000

0

0

Table 1.32

Solution to Exercise 1.4.18 (p. 26)

hundred

thousand

ten thousand million

551,061,300 551,061,000 551,060,000

551,000,000

Table 1.33

Solution to Exercise 1.4.20 (p. 27)

hundred

thousand

ten thousand

million

106,999,413,200 106,999,413,000 106,999,410,000 106,999,000,000

Table 1.34

Solution to Exercise 1.4.22 (p. 27)

Hundred

Thousand ten thousand Million

8,006,000 8,006,000

8,010,000

8,000,000

Table 1.35

Solution to Exercise 1.4.24 (p. 27)

hundred thousand ten thousand million

33,500

33,000

30,000

0

Available for free at Connexions <http://cnx.org/content/col10615/1.4>

Image 107

Image 108

80

CHAPTER 1. ADDITION AND SUBTRACTION OF WHOLE NUMBERS

Table 1.36

Solution to Exercise 1.4.26 (p. 28)

hundred thousand ten thousand million

388,600

389,000

390,000

0

Table 1.37

Solution to Exercise 1.4.28 (p. 28)

hundred thousand ten thousand million

8,200

8,000

10,000

0

Table 1.38

Solution to Exercise 1.4.30 (p. 28)

19,310,000

Solution to Exercise 1.4.32 (p. 29)

29,000,000

Solution to Exercise 1.4.34 (p. 29)

70% or 75%

Solution to Exercise 1.4.36 (p. 29)

$5,500,000,000

Solution to Exercise 1.4.38 (p. 29)

230,000

Solution to Exercise 1.4.40 (p. 29)

5,400,000

Solution to Exercise 1.4.42 (p. 29)

graphing

Solution to Exercise 1.4.44 (p. 29)

Forty-two thousand, one hundred nine

Solution to Exercise 1.4.46 (p. 29)

4,000,000,008

Solution to Exercise 1.5.1 (p. 31)

88

Solution to Exercise 1.5.2 (p. 31)

5,527

Solution to Exercise 1.5.3 (p. 31)

267,166

Available for free at Connexions <http://cnx.org/content/col10615/1.4>

Image 109

Image 110

Image 111

81

Solution to Exercise 1.5.4 (p. 34)

87

= 7 tens + 1 ten + 7 ones

= 8 tens + 7 ones

= 87

Solution to Exercise 1.5.5 (p. 34)

561

= 4 hundreds + 15 tens + 1 ten + 1 one

= 4 hundreds + 16 tens + 1 one

= 4 hundreds + 1 hundred + 6 tens + 1 one

= 5 hundreds + 6 tens + 1 one

= 561

Solution to Exercise 1.5.6 (p. 34)

115

= 10 tens + 1 ten + 5 ones

= 11 tens + 5 ones

= 1 hundred + 1 ten + 5 ones

= 115

Solution to Exercise 1.5.7 (p. 34)

153,525

Solution to Exercise 1.5.8 (p. 34)

167

Solution to Exercise 1.5.9 (p. 35)

2,468

Solution to Exercise 1.5.10 (p. 35)

434,826

Solution to Exercise 1.5.11 (p. 36)

155

Solution to Exercise 1.5.12 (p. 36)

19,750

Solution to Exercise 1.5.13 (p. 36)

30,682

Solution to Exercise 1.5.14 (p. 36)

19

Solution to Exercise 1.5.16 (p. 36)

48

Available for free at Connexions <http://cnx.org/content/col10615/1.4>

82

CHAPTER 1. ADDITION AND SUBTRACTION OF WHOLE NUMBERS

Solution to Exercise 1.5.18 (p. 36)

98

Solution to Exercise 1.5.20 (p. 36)

978

Solution to Exercise 1.5.22 (p. 37)

368

Solution to Exercise 1.5.24 (p. 37)

12,777

Solution to Exercise 1.5.26 (p. 37)

58,738

Solution to Exercise 1.5.28 (p. 37)

45,169,739

Solution to Exercise 1.5.30 (p. 37)

33

Solution to Exercise 1.5.32 (p. 37)

81

Solution to Exercise 1.5.34 (p. 37)

91

Solution to Exercise 1.5.36 (p. 37)

809

Solution to Exercise 1.5.38 (p. 38)

862

Solution to Exercise 1.5.40 (p. 38)

2,321

Solution to Exercise 1.5.42 (p. 38)

1,141,204

Solution to Exercise 1.5.44 (p. 38)

561,364,111

Solution to Exercise 1.5.46 (p. 38)

942,302,364,207,060

Solution to Exercise 1.5.48 (p. 38)

234

Solution to Exercise 1.5.50 (p. 38)

95,365

Solution to Exercise 1.5.52 (p. 39)

1,972,128

Solution to Exercise 1.5.54 (p. 39)

3,700

Solution to Exercise 1.5.56 (p. 39)

3,101,500

Solution to Exercise 1.5.58 (p. 39)

100

Solution to Exercise 1.5.60 (p. 40)

0Solution to Exercise 1.5.62 (p. 40)

1,000

Solution to Exercise 1.5.64 (p. 40)

5Solution to Exercise 1.5.66 (p. 40)

19

Available for free at Connexions <http://cnx.org/content/col10615/1.4>

83

Solution to Exercise 1.5.68 (p. 40)

88

Solution to Exercise 1.5.70 (p. 41)

60,511,000

Solution to Exercise 1.5.72 (p. 41)

5,682,651 square miles

Solution to Exercise 1.5.74 (p. 41)

1,190,000

Solution to Exercise 1.5.76 (p. 42)

271,564,000

Solution to Exercise 1.5.78 (p. 43)

20

Solution to Exercise 1.5.80 (p. 43)

25

Solution to Exercise 1.5.82 (p. 43)

40

Solution to Exercise 1.5.84 (p. 43)

50

Solution to Exercise 1.5.86 (p. 44)

50

Solution to Exercise 1.5.88 (p. 44)

50

Solution to Exercise 1.5.90 (p. 44)

0Solution to Exercise 1.5.92 (p. 44)

6,800

Solution to Exercise 1.6.1 (p. 45)

7 − 5 = 2 since 2 + 5 = 7

Solution to Exercise 1.6.2 (p. 45)

9 − 1 = 8 since 8 + 1 = 9

Solution to Exercise 1.6.3 (p. 45)

17 − 8 = 9 since 9 + 8 = 17

Solution to Exercise 1.6.4 (p. 47)

331

Solution to Exercise 1.6.5 (p. 47)

814

Solution to Exercise 1.6.6 (p. 47)

61,118

Solution to Exercise 1.6.7 (p. 47)

10,000

Solution to Exercise 1.6.8 (p. 47)

62,114

Available for free at Connexions <http://cnx.org/content/col10615/1.4>

Image 112

Image 113

Image 114

84

CHAPTER 1. ADDITION AND SUBTRACTION OF WHOLE NUMBERS

Solution to Exercise 1.6.9 (p. 49)

Solution to Exercise 1.6.10 (p. 49)

Solution to Exercise 1.6.11 (p. 49)

Solution to Exercise 1.6.12 (p. 49)

85

Solution to Exercise 1.6.13 (p. 49)

709

Solution to Exercise 1.6.14 (p. 49)

3,104

Solution to Exercise 1.6.15 (p. 50)

168

Solution to Exercise 1.6.16 (p. 50)

55,640

Solution to Exercise 1.6.17 (p. 50)

1,189

Solution to Exercise 1.6.18 (p. 52)

888

Available for free at Connexions <http://cnx.org/content/col10615/1.4>

85

Solution to Exercise 1.6.19 (p. 52)

4,543

Solution to Exercise 1.6.20 (p. 52)

8,669

Solution to Exercise 1.6.21 (p. 54)

16,134

Solution to Exercise 1.6.22 (p. 54)

4,839

Solution to Exercise 1.6.23 (p. 54)

15,789,940

Solution to Exercise 1.6.24 (p. 55)

4,529

Solution to Exercise 1.6.25 (p. 55)

22,468,227

Solution to Exercise 1.6.26 (p. 55)

7

Solution to Exercise 1.6.28 (p. 55)

6

Solution to Exercise 1.6.30 (p. 55)

3

Solution to Exercise 1.6.32 (p. 55)

41

Solution to Exercise 1.6.34 (p. 55)

209

Solution to Exercise 1.6.36 (p. 56)

21,001

Solution to Exercise 1.6.38 (p. 56)

279,253

Solution to Exercise 1.6.40 (p. 56)

77,472

Solution to Exercise 1.6.42 (p. 56)

24

Solution to Exercise 1.6.44 (p. 56)

188

Solution to Exercise 1.6.46 (p. 56)

2,377

Solution to Exercise 1.6.48 (p. 56)

26,686

Solution to Exercise 1.6.50 (p. 57)

63,143,259

Solution to Exercise 1.6.52 (p. 57)

8,034

Solution to Exercise 1.6.54 (p. 57)

33

Solution to Exercise 1.6.56 (p. 57)

32,611,636,567

Solution to Exercise 1.6.58 (p. 57)

3,938

Solution to Exercise 1.6.60 (p. 57)

8,273,955

Available for free at Connexions <http://cnx.org/content/col10615/1.4>

86

CHAPTER 1. ADDITION AND SUBTRACTION OF WHOLE NUMBERS

Solution to Exercise 1.6.62 (p. 58)

51

Solution to Exercise 1.6.64 (p. 58)

3,405

Solution to Exercise 1.6.66 (p. 58)

26

Solution to Exercise 1.6.68 (p. 58)

72,069

Solution to Exercise 1.6.70 (p. 58)

3,197

Solution to Exercise 1.6.72 (p. 58)

29

Solution to Exercise 1.6.74 (p. 58)

10,385

Solution to Exercise 1.6.76 (p. 58)

15%

Solution to Exercise 1.6.78 (p. 58)

11,247,000

Solution to Exercise 1.6.80 (p. 59)

436

Solution to Exercise 1.6.82 (p. 59)

57,187,000

Solution to Exercise 1.6.84 (p. 59)

165,000

Solution to Exercise 1.6.86 (p. 59)

74

Solution to Exercise 1.6.88 (p. 60)

4,547

Solution to Exercise 1.6.90 (p. 60)

11

Solution to Exercise 1.6.92 (p. 61)

10

Solution to Exercise 1.6.94 (p. 61)

12

Solution to Exercise 1.6.96 (p. 61)

76

Solution to Exercise 1.6.98 (p. 61)

51

Solution to Exercise 1.6.100 (p. 61)

60

Solution to Exercise 1.6.102 (p. 61)

165

Solution to Exercise 1.6.104 (p. 61)

214

Solution to Exercise 1.6.106 (p. 62)

330 (answers may vary)

Solution to Exercise 1.6.108 (p. 62)

27,000,000

Solution to Exercise 1.7.1 (p. 63)

12 + 41 = 53 and 41 + 12 = 53

Available for free at Connexions <http://cnx.org/content/col10615/1.4>

87

Solution to Exercise 1.7.2 (p. 63)

837 + 1, 958 = 2, 795 and 1, 958 + 837 = 2, 795

Solution to Exercise 1.7.3 (p. 63)

(17 + 32) + 25 = 49 + 25 = 74 and 17 + (32 + 25) = 17 + 57 = 74

Solution to Exercise 1.7.4 (p. 63)

(1, 629 + 806) + 429 = 2, 435 + 429 = 2, 864

1, 629 + (806 + 429) = 1, 629 + 1, 235 = 2, 864

Solution to Exercise 1.7.5 (p. 64)

8

Solution to Exercise 1.7.6 (p. 64)

5Solution to Exercise 1.7.7 (p. 64)

xSolution to Exercise 1.7.8 (p. 64)

xSolution to Exercise 1.7.9 (p. 65)

17

Solution to Exercise 1.7.10 (p. 65)

37

Solution to Exercise 1.7.12 (p. 65)

45

Solution to Exercise 1.7.14 (p. 65)

568

Solution to Exercise 1.7.16 (p. 65)

122,323

Solution to Exercise 1.7.18 (p. 66)

45

Solution to Exercise 1.7.20 (p. 66)

100

Solution to Exercise 1.7.22 (p. 66)

556

Solution to Exercise 1.7.24 (p. 66)

43,461

Solution to Exercise 1.7.26 (p. 66)

132 + 6 =80 + 58 = 138

Solution to Exercise 1.7.28 (p. 67)

987 + 171 =731 + 427 = 1, 158

Solution to Exercise 1.7.30 (p. 67)

Identity

Solution to Exercise 1.7.32 (p. 67)

15 + 8 = 8 + 15 = 23

Solution to Exercise 1.7.34 (p. 67)

. . .because its partner in addition remains identically the same after that addition Solution to Exercise 1.7.36 (p. 67)

Two thousand, two hundred eighteen.

Solution to Exercise 1.7.38 (p. 67)

550

Solution to Exercise 1.9.1 (p. 69)

937

Solution to Exercise 1.9.3 (p. 70)

565

Available for free at Connexions <http://cnx.org/content/col10615/1.4>

88

CHAPTER 1. ADDITION AND SUBTRACTION OF WHOLE NUMBERS

Solution to Exercise 1.9.5 (p. 70)

559

Solution to Exercise 1.9.7 (p. 70)

1,342

Solution to Exercise 1.9.9 (p. 70)

2,001

Solution to Exercise 1.9.11 (p. 70)

1,963

Solution to Exercise 1.9.13 (p. 70)

79,456

Solution to Exercise 1.9.15 (p. 70)

96,953

Solution to Exercise 1.9.17 (p. 71)

791,824

Solution to Exercise 1.9.19 (p. 71)

8,301

Solution to Exercise 1.9.21 (p. 71)

140,381

Solution to Exercise 1.9.23 (p. 71)

76,224

Solution to Exercise 1.9.25 (p. 71)

4,955

Solution to Exercise 1.9.27 (p. 71)

185,611

Solution to Exercise 1.9.29 (p. 72)

2,238

Solution to Exercise 1.9.31 (p. 72)

1,338

Solution to Exercise 1.9.33 (p. 72)

878

Solution to Exercise 1.9.35 (p. 72)

618,227

Solution to Exercise 1.9.37 (p. 72)

1,621

Solution to Exercise 1.9.39 (p. 73)

484,601

Solution to Exercise 1.9.41 (p. 73)

19,853

Solution to Exercise 1.9.43 (p. 73)

1,702

Solution to Exercise 1.9.45 (p. 73)

1,114

Solution to Exercise 1.9.47 (p. 73)

1,300

Solution to Exercise 1.9.49 (p. 73)

2,718

Solution to Exercise 1.9.51 (p. 73)

7,356

Solution to Exercise 1.9.53 (p. 74)

3,415

Available for free at Connexions <http://cnx.org/content/col10615/1.4>

Image 115

89

Solution to Exercise 1.9.55 (p. 74)

11,827

Solution to Exercise 1.9.57 (p. 74)

407,262

Solution to Exercise 1.9.59 (p. 74)

718,478

Solution to Exercise 1.9.61 (p. 74)

9,941

Solution to Exercise 1.9.63 (p. 74)

626 + 1, 242 = 1, 242 + 626 = 1, 868

Solution to Exercise 1.10.1 (p. 74)

9Solution to Exercise 1.10.2 (p. 74)

ones, tens, hundreds

Solution to Exercise 1.10.3 (p. 74)

8Solution to Exercise 1.10.4 (p. 74)

no

Solution to Exercise 1.10.5 (p. 74)

Solution to Exercise 1.10.6 (p. 75)

Sixty-three thousand, four hundred twenty-ve

Solution to Exercise 1.10.7 (p. 75)

18,359,072

Solution to Exercise 1.10.8 (p. 75)

400

Solution to Exercise 1.10.9 (p. 75)

19,000

Solution to Exercise 1.10.10 (p. 75)

500

Solution to Exercise 1.10.11 (p. 75)

675

Solution to Exercise 1.10.12 (p. 75)

4,027

Solution to Exercise 1.10.13 (p. 75)

188

Solution to Exercise 1.10.14 (p. 75)

23,501

Solution to Exercise 1.10.15 (p. 75)

90

Solution to Exercise 1.10.16 (p. 75)

304

Solution to Exercise 1.10.17 (p. 75)

70,123

Solution to Exercise 1.10.18 (p. 75)

391

Solution to Exercise 1.10.19 (p. 75)

182

Solution to Exercise 1.10.20 (p. 75)

Yes, commutative property of addition

Available for free at Connexions <http://cnx.org/content/col10615/1.4>

90

CHAPTER 1. ADDITION AND SUBTRACTION OF WHOLE NUMBERS

Available for free at Connexions <http://cnx.org/content/col10615/1.4>

Chapter 2

Multiplication and Division of Whole

Numbers

2.1 Objectives1

After completing this chapter, you should

Multiplication of Whole Numbers (Section 2.2)

• understand the process of multiplication

• be able to multiply whole numbers

• be able to simplify multiplications with numbers ending in zero

• be able to use a calculator to multiply one whole number by another

Concepts of Division of Whole Numbers (Section 2.3)

• understand the process of division

• understand division of a nonzero number into zero

• understand why division by zero is undened

• be able to use a calculator to divide one whole number by another

Division of Whole Numbers (Section 2.4)

• be able to divide a whole number by a single or multiple digit divisor

• be able to interpret a calculator statement that a division results in a remainder Some Interesting Facts about Division (Section 2.5)

• be able to recognize a whole number that is divisible by 2, 3, 4, 5, 6, 8, 9, or 10

Properties of Multiplication (Section 2.6)

• understand and appreciate the commutative and associative properties of multiplication

• understand why 1 is the multiplicative identity

1This content is available online at <http://cnx.org/content/m18888/1.4/>.

Available for free at Connexions <http://cnx.org/content/col10615/1.4> 91

92

CHAPTER 2. MULTIPLICATION AND DIVISION OF WHOLE NUMBERS

2.2 Multiplication of Whole Numbers2

2.2.1 Section Overview

• Multiplication

• The Multiplication Process With a Single Digit Multiplier

• The Multiplication Process With a Multiple Digit Multiplier

• Multiplication With Numbers Ending in Zero

• Calculators

2.2.2 Multiplication

Multiplication is a description of repeated addition.

In the addition of

5 + 5 + 5

the number 5 is repeated 3 times. Therefore, we say we have three times ve and describe it by writing 3 × 5

Thus,

3 × 5 = 5 + 5 + 5

Multiplicand

In a multiplication, the repeated addend (number being added) is called the multiplicand. In 3 × 5, the 5

is the multiplicand.

Multiplier

Also, in a multiplication, the number that records the number of times the multiplicand is used is called the multiplier. In 3 × 5, the 3 is the multiplier.

2.2.2.1 Sample Set A

Express each repeated addition as a multiplication. In each case, specify the multiplier and the multiplicand.

Example 2.1

7 + 7 + 7 + 7 + 7 + 7

6 × 7.

Multiplier is 6.

Multiplicand is 7.

Example 2.2

18 + 18 + 18

3 × 18.

Multiplier is 3.

Multiplicand is 18.

2This content is available online at <http://cnx.org/content/m34863/1.2/>.

Available for free at Connexions <http://cnx.org/content/col10615/1.4>

Image 116

93

2.2.2.2 Practice Set A

Express each repeated addition as a multiplication. In each case, specify the multiplier and the multiplicand.

Exercise 2.2.1

(Solution on p. 142.)

12 + 12 + 12 + 12

. Multiplier is

. Multiplicand is

.

Exercise 2.2.2

(Solution on p. 142.)

36 + 36 + 36 + 36 + 36 + 36 + 36 + 36

. Multiplier is

. Multiplicand is

.

Exercise 2.2.3

(Solution on p. 142.)

0 + 0 + 0 + 0 + 0

. Multiplier is

. Multiplicand is

.

Exercise 2.2.4

(Solution on p. 142.)

1847 + 1847 + … + 1847

|

{z

}

12, 000 times

. Multiplier is

. Multiplicand is

.

Factors

In a multiplication, the numbers being multiplied are also called factors.

Products

The result of a multiplication is called the product. In 3 × 5 = 15, the 3 and 5 are not only called the multiplier and multiplicand, but they are also called factors. The product is 15.

Indicators of Multiplication ×,·,( )

The multiplication symbol (×) is not the only symbol used to indicate multiplication. Other symbols include the dot ( · ) and pairs of parentheses ( ). The expressions

3 × 5,

3 · 5,

3 (5),

(3) 5,

(3) (5)

all represent the same product.

2.2.3 The Multiplication Process With a Single Digit Multiplier

Since multiplication is repeated addition, we should not be surprised to notice that carrying can occur.

Carrying occurs when we nd the product of 38 and 7:

First, we compute 7 × 8 = 56. Write the 6 in the ones column. Carry the 5. Then take 7 × 3 = 21. Add to 21 the 5 that was carried: 21 + 5 = 26. The product is 266.

Available for free at Connexions <http://cnx.org/content/col10615/1.4>

Image 117

Image 118

Image 119

94

CHAPTER 2. MULTIPLICATION AND DIVISION OF WHOLE NUMBERS

2.2.3.1 Sample Set B

Find the following products.

Example 2.3

3 × 4 = 12

Write the 2, carry the 1.

3 × 6 = 18

Add to 18 the 1 that was carried: 18 + 1 = 19.

The product is 192.

Example 2.4

5 × 6 = 30

Write the 0, carry the 3.

5 × 2 = 10

Add to 10 the 3 that was carried: 10 + 3 = 13. Write the 3, carry the 1.

5 × 5 = 25

Add to 25 the 1 that was carried: 25 + 1 = 6.

The product is 2,630.

Example 2.5

9 × 4 = 36

Write the 6, carry the 3.

9 × 0 = 0

Add to the 0 the 3 that was carried: 0 + 3 = 3. Write the 3.

9 × 8 = 72

Write the 2, carry the 7.

Add to the 9 the 7 that was carried: 9 + 7 = 16.

9 × 1 = 9

Since there are no more multiplications to perform,write both the 1 and 6.

The product is 16,236.

2.2.3.2 Practice Set B

Find the following products.

Exercise 2.2.5

(Solution on p. 142.)

37

× 5

Available for free at Connexions <http://cnx.org/content/col10615/1.4>

Image 120

Image 121

95

Exercise 2.2.6

(Solution on p. 142.)

78

× 8

Exercise 2.2.7

(Solution on p. 142.)

536

×

7

Exercise 2.2.8

(Solution on p. 142.)

40, 019

×

8

Exercise 2.2.9

(Solution on p. 142.)

301, 599

×

3

2.2.4 The Multiplication Process With a Multiple Digit Multiplier

In a multiplication in which the multiplier is composed of two or more digits, the multiplication must take place in parts. The process is as follows:

Part 1: First Partial Product Multiply the multiplicand by the ones digit of the multiplier. This product is called the rst partial product.

Part 2: Second Partial Product Multiply the multiplicand by the tens digit of the multiplier. This product is called the second partial product. Since the tens digit is used as a factor, the second partial product is written below the rst partial product so that its rightmost digit appears in the tens column.

Part 3: If necessary, continue this way nding partial products. Write each one below the previous one so that the rightmost digit appears in the column directly below the digit that was used as a factor.

Part 4: Total Product Add the partial products to obtain the total product.

note: It may be necessary to carry when nding each partial product.

2.2.4.1 Sample Set C

Example 2.6

Multiply 326 by 48.

Part 1:

License

Math Copyright © by sboschman. All Rights Reserved.

Share This Book